Cardiovascular-health problem

Ace your homework & exams now with Quizwiz!

A client with a history of cardiac disease is due for a morning dose of furosemide. Which serum potassium level, if noted in the client's laboratory report, would be reported before administering the dose of furosemide? 1. 3.2 mEq/L (3.2 mmol/L) 2. 3.8 mEq/L (3.8 mmol/L) 3. 4.2 mEq/L (4.2 mmol/L) 4. 4.8 mEq/L (4.8 mmol/L)

correct answer 1. 3.2 mEq/L (3.2 mmol/L) Rationale: The normal serum potassium level in the adult is 3.5 mEq/L to 5.0 mEq/L (3.5-5.0 mmol/L). The correct option is the only value that falls below the therapeutic range. Administering furosemide to a client with a low potassium level and a history of cardiac problems could precipitate ventricular dysrhythmias. The remaining options are within the normal range.

A client who had a myocardial infarction is at risk for cardiogenic shock. The nurse assisting in planning care understands that the primary cause of cardiogenic shock results from which process? 1. A pump failure and reduction in cardiac output 2. A physical obstruction that decreases filling or outflow of blood 3. Dilated vasculature decreasing the movement of blood to the body 4. Loss of vasoconstrictor tone leading to pooling of blood in vessels

correct answer 1. A pump failure and reduction in cardiac output Rationale: Cardiogenic shock is caused by the inability of the heart itself to pump effectively, resulting in decreased cardiac output. Cardiac output reflects blood reaching the tissues and vasculature. The remaining options do not describe the pathophysiology associated with cardiogenic shock.

About which laboratory values would the nurse be immediately concerned because they could cause cardiac arrest? Select all that apply. 1. Calcium 7.9 mg/L 2Potassium 5.9 mEq/L 3Sodium 138 mEq/L 4Chloride 104 mEq/L 5Magnesium 1.7 mg/dL

correct answer 1. Calcium 7.9 mg/L 2Potassium 5.9 mEq/L Rationale: Normal laboratory values are calcium, 9.0 to 10.5 mg/dL; sodium, 135 to 145 mEq/L; potassium, 3.5 to 5.0 mEq/L; chloride, 98 to 106 mEq/L; and magnesium 1.8 to 2.6 mg/dL. Decreased calcium and/or increased potassium can lead to cardiac arrest

A licensed practical nurse (LPN) is assisting in the care of a client who is having central venous pressure (CVP) measurements taken by the registered nurse (RN). The LPN would assist the RN by placing the bed in which position for the reading? 1. Flat 2. Semi-Fowler's 3. Trendelenburg's 4. Reverse Trendelenburg's

correct answer 1. Flat Rationale: To obtain a CVP measurement, the head of the bed should be flat in order for the readings to be accurate. The use of the other positions listed would result in false low or false high readings.

The nurse notes that a client is being treated with nesiritide. The nurse would expect this client to be experiencing which disorder? 1. Heart failure 2 Polycythemia 3. Hepatic failure 4. Myocardial infarctio

correct answer 1. Heart failure Rationale: Nesiritide is a synthetic form of human B-type natriuretic peptide (BNP) indicated only for short-term, intravenous therapy of hospitalized clients with acutely decompensated heart failure. It is not used for polycythemia, hepatic failure, or myocardial infarction.

The nurse is reviewing laboratory results of a digoxin level for the client taking digoxin. The digoxin level is 2.5 ng/mL, which indicates digoxin toxicity. Which signs and symptoms would the nurse note? Select all that apply. 1. Nausea 2. Syncope 3. Polyphagia 4. Bradycardia 5. Constipation

correct answer 1. Nausea 2. Syncope 4. Bradycardia Rationale: Digoxin has a narrow therapeutic range. The therapeutic range is 0.8 to 2 ng/mL. Drug levels higher than the therapeutic level greatly increase the risk of toxicity. Classic symptoms of digitalis toxicity are yellow-green halos around lights, nausea, diarrhea, and confusion

The nurse is planning measures to decrease the incidence of chest pain for a client with angina pectoris. The nurse would do which intervention to effectively accomplish this goal? 1. Provide a quiet and low-stimulus environment. 2. Encourage the family to come visit very frequently. 3. Encourage the client to call friends and relatives each day. 4. Recommend that the client watch TV as a constant diversion.

correct answer 1. Provide a quiet and low-stimulus environment. Rationale: Chest pain can be minimized by a quiet, low-stimulus environment, which reduces factors that trigger chest pain such as emotional excitement. Each of the incorrect options increases the amount of client stimulation, which increases the risk of an anginal episode.

The nurse has completed diet teaching for a client who has been prescribed a low-sodium diet to treat hypertension. The nurse determines that there is a need for further teaching when the client makes which statement? 1. "This diet will help lower my blood pressure." 2. "Fresh foods such as fruits and vegetables are high in sodium." 3."This diet is not a replacement for my antihypertensive medications." 4. "The reason I need to lower my salt intake is to reduce fluid retention."

correct answer 2. "Fresh foods such as fruits and vegetables are high in sodium." Rationale: A low-sodium diet is used as an adjunct to antihypertensive medications for the treatment of hypertension. Sodium retains fluid, which leads to hypertension secondary to increased fluid volume. Fresh foods such as fruits and vegetables are low in sodium.

The nurse volunteering at the health screening clinic reinforces instructions to a 22-year-old client that diet and exercise would be used as tools to keep the total cholesterol level under at least which level? 1. 130 mg/dL 2. 200 mg/dL 3. 250 mg/dL 4. 300 mg/dL

correct answer 2. 200 mg/dL Rationale: The cholesterol level should be at least less than 199 mg/dL. The client should be counseled to keep the total cholesterol level under 200 mg/dL. This will aid in prevention of atherosclerosis, which can lead to a number of cardiovascular disorders later in life. The option of 130 mg/dL is abnormally low, and the options of 250 mg/dL and 300 mg/dL are too high.

A client with heart disease is instructed regarding a low-fat diet. The nurse determines that the client understands the diet if the client states to avoid which food item? 1. Apples 2. Cheese 3. Oranges 4. Skim milk

correct answer 2. Cheese Rationale:Fruits, vegetables, and skim milk contain minimal amounts of fat. Cheese is high in fat.

A client arrives at the health care clinic for follow-up care and evaluation of the effectiveness of prazosin. Which finding indicates a therapeutic effect related to the use of this medication? 1. Increased platelet count 2. Decrease in blood pressure 3. Increased red blood cell count 4. Decrease in blood glucose level

correct answer 2. Decrease in blood pressure Rationale: Prazosin is an antihypertensive medication. The principal indication for its use is hypertension. A decrease in blood pressure indicates a therapeutic effect of the medication. Options 1, 3, and 4 are unrelated to the use of this medication.

An older client with ischemic heart disease has experienced an episode of dizziness and shortness of breath. The nurse reviews the plan of care and notices documentation of decreased cardiac output, dyspnea, and syncopal episodes. The nurse plans to take which important action? 1. Monitor oxygen saturation levels. 2. Place the client on a cardiac monitor. 3. Measure blood pressure every 4 hours. 4. Check capillary refill at least once per shift.

correct answer 2. Place the client on a cardiac monitor. Rationale: The client with decreased cardiac output should be placed on continuous cardiac monitoring so myocardial perfusion and presence of dysrhythmias can be most accurately assessed. Other cardiovascular data should be collected at least every 2 hours initially.

The nurse walks into a client's room and notes ventricular tachycardia on the client's bedside monitor. The nurse is anticipating needing to defibrillate this rhythm. What other rhythms would require defibrillation? 1. Asystole 2. Ventricular fibrillation 3. Supraventricular tachycardia 4. Pulseless electrical activity (PEA)

correct answer 2. Ventricular fibrillation Rationale: Ventricular fibrillation and ventricular tachycardia are pulseless rhythms that can be defibrillated. Non-shockable rhythms include PEA and asystole. Supraventricular tachycardia is a rhythm that requires cardioversion and not defibrillation.

A client is admitted to the hospital with possible rheumatic endocarditis. The nurse would check for a history of which type of infection? 1. Viral infection 2. Yeast infection 3. Streptococcal infection 4. Staphylococcal infection

correct answer 3. Streptococcal infection Rationale: Rheumatic endocarditis, also called rheumatic carditis, is a major indicator of rheumatic fever, which is a complication of infection with group A beta-hemolytic streptococcal infections. It is frequently triggered by streptococcal pharyngitis. Options 1, 2, and 4 are incorrect.

Isosorbide mononitrate is prescribed for a client with angina pectoris. The client tells the nurse that the medication is causing a chronic headache. Which action would the nurse suggest to the client? 1. Cut the dose in half. 2. Discontinue the medication. 3. Take the medication with food. 4. Contact the primary health care provider (PHCP).

correct answer 3. Take the medication with food. Rationale: Isosorbide mononitrate is an antianginal medication. Headache is a frequent side effect of isosorbide mononitrate and usually disappears during continued therapy. If a headache occurs during therapy, the client should be instructed to take the medication with food or meals. It is not necessary to contact the PHCP unless the headaches persist with therapy. It is not appropriate to instruct the client to discontinue therapy or adjust the dosages.

A client with heart failure is scheduled to be discharged to home with digoxin and furosemide as ongoing prescribed medications. The nurse teaches the client to report which sign/symptom that indicates the medications are not producing the intended effect? 1. Decrease in pedal edema 2. High urine output during the day 3. Weight gain of 2 to 3 pounds in a few days 4. Cough accompanied by other signs of respiratory infection

correct answer 3. Weight gain of 2 to 3 pounds in a few days Rationale: Clients with heart failure should immediately report weight gain, loss of appetite, shortness of breath with activity, edema, persistent cough, and nocturia. An increase in daytime voiding is expected while on diuretic therapy. A cough as a result of respiratory infection does not necessarily indicate that heart failure is exacerbating.

A client diagnosed with angina pectoris returns to the nursing unit after experiencing an angioplasty. The nurse reinforces instructions to the client regarding the procedure and home care measures. Which statement by the client indicates an understanding of the instructions? 1. "I am considering cutting my workload." 2. "I need to cut down on cigarette smoking." 3. "I am so relieved that my heart is repaired." 4. "I need to adhere to my dietary restrictions."

correct answer 4. "I need to adhere to my dietary restrictions." Rationale: Following the angioplasty, the client needs to be instructed about specific dietary restrictions that must be followed. Following the recommended dietary and lifestyle changes helps prevent further atherosclerosis. Abrupt closure of the artery can occur if the recommended dietary and lifestyle changes are not followed. Cigarette smoking needs to be stopped. An angioplasty does not repair the heart.

A client is taking ticlopidine hydrochloride. The nurse tells the client to avoid which substance while taking this medication? 1. Vitamin C 2. Vitamin D 3. Acetaminophen 4. Acetylsalicylic acid

correct answer 4. Acetylsalicylic acid Rationale: Ticlopidine hydrochloride is a platelet aggregation inhibitor. It is used to decrease the risk of thrombotic strokes in clients with precursor symptoms. Because it is an antiplatelet agent, other medications that precipitate or aggravate bleeding should be avoided during its use. Therefore, aspirin or any aspirin-containing product should be avoided.

The nurse walks into a client's room and notes what appears to be ventricular tachycardia on the monitor. The nurse cannot find a palpable pulse. After beginning cardiopulmonary resuscitation (CPR), what steps would the nurse take next? 1. Attach the defibrillator and shock 2. Call for help and wait until help arrives 3. Attach the defibrillator and establish intravenous access 4. Attach the defibrillator and determine if the rhythm is shockable

correct answer 4. Attach the defibrillator and determine if the rhythm is shockable Rationale: Beginning CPR on a fatal rhythm is the primary need for the client. Once CPR has begun, attaching the client to a monitor to determine if the rhythm is shockable is the next step. CPR can continue while intravenous access is established, while medications are administered, and while reversible causes are discussed, but if the rhythm can be defibrillated, it must be determined as one of the initial actions.

A client who has developed atrial fibrillation that is not responding to medication therapy has begun taking warfarin. The nurse is reinforcing dietary discharge teaching with the client. The nurse would plan to teach the client to avoid which food while taking this medication? 1. Cherries 2. Potatoes 3. Spaghetti 4. Broccoli

correct answer 4. Broccoli Rationale: Anticoagulant medications work by antagonizing the action of vitamin K, which is needed for clotting. When a client is taking an anticoagulant, foods high in vitamin K often are omitted from the diet. Vitamin K is found in large amounts in green leafy vegetables such as broccoli, cabbage, turnip greens, and lettuce. The other options listed are foods that are lower in vitamin K.

The nurse is visiting an older client whose family has gone out for the day. During the visit, the client experiences chest pain that is unrelieved by nitroglycerin given by the nurse. Which action by the nurse would be appropriate at this time? 1. Notify a family member who is the next of kin. 2. Drive the client to the primary health care provider's office. 3. Inform the home care agency supervisor that the visit may be prolonged. 4. Call for an ambulance to transport the client to the emergency department.

correct answer 4. Call for an ambulance to transport the client to the emergency department. Rationale: Chest pain that is unrelieved by rest and nitroglycerin may not be typical anginal pain but may signal myocardial infarction (MI). Because the risk of sudden cardiac death is greatest in the first 24 hours after MI, it is imperative that the client receive emergency cardiac care. A primary health care provider's office is not equipped to treat MI. Communication with the family or home care agency delays client treatment, which is needed immediately.

An adult client just admitted to the hospital with heart failure also has a history of diabetes mellitus. The nurse consults with the registered nurse to verify a prescription for which medication that the client was taking before admission? 1. Acarbose 2. NPH insulin 3. Regular insulin 4. Chlorpropamide

correct answer 4. Chlorpropamide Rationale: Chlorpropamide is an oral hypoglycemic agent that exerts an antidiuretic effect and should be administered cautiously or avoided in the client with cardiac impairment or fluid retention. It is a first-generation sulfonylurea. Insulin does not cause or aggravate fluid retention. Acarbose is a miscellaneous oral hypoglycemic agent.

A client who has received sodium bicarbonate in large amounts is at risk for developing metabolic alkalosis. The nurse checks this client for which signs and symptoms characteristic of this disorder? 1. Disorientation and dyspnea 2. Drowsiness, headache, and tachypnea 3. Tachypnea, dizziness, and paresthesias 4. Decreased respiratory depth and rate and dysrhythmias

correct answer 4. Decreased respiratory depth and rate and dysrhythmias Rationale: The client with metabolic alkalosis is likely to exhibit a decrease in respiratory rate and depth, nausea, vomiting, diarrhea, restlessness, numbness and tingling in the extremities, twitching in the extremities, hypokalemia, hypocalcemia, and dysrhythmias. Disorientation and dyspnea could be associated with hypoxemia. Tachypnea, dizziness, and paresthesias are often associated with hyperventilation and respiratory alkalosis. Drowsiness, headache and tachypnea are not associated with metabolic alkalosis.

The nurse is assisting in monitoring the condition of a client after pericardiocentesis for cardiac tamponade. Which observation indicates that the procedure was unsuccessful? 1. Clear breath sounds 2. Client expressions of relief 3. Clearly audible heart sounds 4. Distant and muffled heart sounds

correct answer 4. Distant and muffled heart sounds Rationale: Following pericardiocentesis, the client usually expresses immediate relief. Heart sounds are no longer muffled or distant. Clear breath sounds and clearly audible heart sounds are positive signs.

A client taking an aldosterone antagonist known as eplerenone for hypertension asks about side effects of this medication. The nurse tells the client that which sign/symptom is a side effect? 1. Coughing 2. Rash and urticaria 3. Headache and dizziness 4. Elevated potassium level

correct answer 4. Elevated potassium level Rationale: Eplerenone is a selective aldosterone receptor blocker. The medication is used for hypertension and heart failure and has one significant side effect, hyperkalemia. Therefore, options 1, 2, and 3 are incorrect.

The nurse enters a client's room to check the client who began receiving a blood transfusion 45 minutes earlier. The client is flushed and dyspneic. The nurse listens to the client's lung sounds and notes the presence of crackles in the lung bases. The nurse determines that this client is most likely experiencing which complication of blood transfusion therapy? 1. Bacteremia 2. Hypovolemic shock 3. Transfusion reaction 4. Fluid (circulatory) overload

correct answer 4. Fluid (circulatory) overload Rationale: With fluid (circulatory) overload, the client has the presence of crackles in the lungs in addition to dyspnea. Hypovolemic shock (restlessness, increased pulse, decreased blood pressure) is not likely to occur in a client receiving fluids. An allergic reaction, which is one type of blood transfusion reaction, would produce symptoms such as flushing, dyspnea, itching, and a generalized rash. With bacteremia, the client would have a fever, which is not part of the clinical picture presented.

The nurse is assisting a hospitalized client who is newly diagnosed with coronary artery disease (CAD) to make appropriate selections from the dietary menu. The nurse encourages the client to select which meal? 1. Sausage, pancakes, and toast 2. Broccoli, buttered rice, and grilled chicken 3. Hamburger, baked apples, and avocado salad 4. Fresh strawberries, steamed vegetables, and baked fish

correct answer 4. Fresh strawberries, steamed vegetables, and baked fish Rationale: Diets high in saturated fats raise the serum lipid level, which in turn raises the blood cholesterol. Over time, high blood cholesterol levels lead to the development of atherosclerosis and diseases such as coronary artery disease. A diet that is low in saturated fats is helpful in reducing the progression of atherosclerosis. Meats and dairy products tend to be higher in fat than other food groups.

The nurse carries out a standard prescription for a stat electrocardiogram (ECG) on a client who has an episode of chest pain. The nurse would take which action next? 1. Do a repeat 12-lead ECG. 2. Wait to see whether the pain resolves. 3. Report the episode of chest pain to the primary health care provider. 4. Give sublingual nitroglycerin per the primary health care provider's prescriptions.

correct answer 4. Give sublingual nitroglycerin per the primary health care provider's prescriptions. Rationale: After completing the stat ECG, the nurse would administer a nitroglycerin tablet to dilate the coronary arteries and relieve ischemic pain. The nurse would not wait to see whether pain resolves on its own but would determine whether the pain is relieved with nitroglycerin. The nurse would do a repeat ECG if it is prescribed. The nurse would report the episode of pain to the primary health care provider but should administer the nitroglycerin before doing so.

A client seeks medical attention for intermittent episodes in which the fingers of both hands become cold, pale, and numb. The client states that they then become reddened and swollen with a throbbing, achy pain and Raynaud's disease is diagnosed. Which factor would precipitate these episodes? 1. Exposure to heat 2. Being in a relaxed environment 3. Prolonged episodes of inactivity 4. Ingestion of coffee or chocolate

correct answer 4. Ingestion of coffee or chocolate Rationale: Raynaud's disease is a bilateral form of intermittent arteriolar spasm, which can be classified as obstructive or vasospastic. Episodes are characterized by pallor, cold, numbness, and possible cyanosis followed by erythema, tingling, and aching pain in the fingers. Attacks are triggered by exposure to cold, nicotine, caffeine, trauma to the fingertips, and stress.

A client in a long-term care facility who has a history of angina pectoris wants to go for a short walk outside with a family member. It is a sunny but chilly December day. The nurse would perform which intervention to care for this client in a holistic manner? 1. Tell the client that this is not allowed. 2. Tell the family member not to take the client outdoors. 3. Give the client a cup of hot coffee before going outside. 4. Instruct the family member to dress the client warmly before going outside.

correct answer 4. Instruct the family member to dress the client warmly before going outside. Rationale: The nurse should meet both the physiological and psychosocial needs of the client in a holistic manner by asking the family member to be sure that the client is dressed warmly before going outside. Option 4 is correct because dressing the client warmly will decrease the chance of vasoconstriction, which may lead to an angina attack. Options 1 and 2 ignore the psychosocial needs. Option 3 is detrimental to physiological needs because, in addition to the cold weather, caffeine places an additional burden on the heart.

A client is receiving a continuous intravenous infusion of heparin sodium to treat deep vein thrombosis. The client's activated partial thromboplastin (aPTT) time is 65 seconds. The licensed practical nurse reviews the laboratory results with the registered nurse, anticipating that which action is needed? 1. Discontinuing the heparin infusion 2. Increasing the rate of the heparin infusion 3. Decreasing the rate of the heparin infusion 4. Leaving the rate of the heparin infusion as is

correct answer 4. Leaving the rate of the heparin infusion as is Rationale: The normal aPTT varies between 28 seconds and 35 seconds, depending on the type of activator used in testing. The therapeutic dose of heparin for treatment of deep vein thrombosis is to keep the aPTT between 1.5 and 2.5 times normal. This means that the client's value should not be less than 40 seconds or greater than 87.5 seconds. Thus the client's aPTT is within the therapeutic range, and the dose should remain unchanged.

The nurse is caring for a client with a diagnosis of myocardial infarction (MI) and is assisting the client in completing the diet menu. Which beverage does the nurse instruct the client to select from the menu? 1. Tea 2. Cola 3. Coffee 4. Lemonade

correct answer 4. Lemonade Rationale: A client with a diagnosis of MI should not consume caffeinated beverages. Caffeinated products can produce a vasoconstrictive effect leading to further cardiac ischemia. Coffee, tea, and cola all contain caffeine and need to be avoided in the client with MI.

The nurse is discharging a client from the hospital who was given a prescription for atorvastatin. The nurse would tell the client to report which adverse effect to the primary health care provider immediately? 1. Tiredness and fatigue 2. Flushing and redness 3. Flatulence and constipation 4. Muscle pain and weakness

correct answer 4. Muscle pain and weakness Rationale: Atorvastatin can injure muscle tissue. The muscle injury can progress to myositis, which is muscle inflammation associated with moderate elevation of creatine kinase (CK), an enzyme released from injured muscle. Myositis, in turn, may progress to potentially fatal rhabdomyolysis, defined as muscle disintegration or dissolution associated with a marked elevation of CK and possibly with renal failure. Tiredness and fatigue and flushing and redness are not related to atorvastatin therapy. Muscle pain and weakness is an expected side effect.

An adult client presents to the emergency department (ED) with complaints of substernal chest pain and is taken directly to a stretcher in the department. Which priority intervention would the nurse undertake? 1. Check vital signs. 2. Check the pulse oximetry. 3. Connect the client to a monitor. 4. Obtain an electrocardiogram (ECG).

correct answer 4. Obtain an electrocardiogram (ECG). Rationale: As long as a client is alert and breathing, an ECG is the first priority when a client presents to the ED with chest pain. The paper copy of the ECG needs to be reviewed immediately by the primary health care provider. Then the client is connected to a cardiac monitor, vital signs are obtained, and a brief discussion about a description and rating of the pain is done. Oxygen saturations are checked while connecting to nasal O2 at 4 to 6 L/minute. An 81-mg chewable aspirin is given to the alert client if applicable, and nitroglycerin tablets grains 1/150 sublingual are administered with the primary health care provider's approval.

The nurse is caring for a client who is anxious and is experiencing dyspnea and restlessness from hypoxemia associated with pulmonary edema. Auscultation of the lungs reveals these breath sounds. (Refer to audio.) The nurse determines that these breath sounds usually result from which cause? 1. Obstruction of the bronchus 2. Inflammation of the pleural surfaces 3. Passage of air through a narrowed airway 4. Opening of small airways that contain fluid

correct answer 4. Opening of small airways that contain fluid Rationale: The sounds that the nurse hears are high-pitched crackles. Crackles are audible when there is a sudden opening of small airways that contain fluid. Crackles are usually heard during inspiration and do not clear with a cough. They resemble the sound of a lock of hair being rubbed between the thumb and forefinger and are heard in conditions such as pulmonary edema. High-pitched crackles are characteristically fine and are high-pitched discontinuous popping noises (nonmusical sounds) heard during the end of inspiration. Medium-pitched crackles produce a moist sound about halfway through inspiration. Coarse crackles are low-pitched bubbling sounds that start early in inspiration and extend into the first part of expiration. Rhonchi (low-pitched, coarse, loud, low snoring or moaning sounds) are heard in conditions causing obstruction of the bronchus or trachea. A pleural friction rub (a superficial low-pitched coarse rubbing or grating sound) is heard when the pleural surfaces are inflamed. Passage of air through a narrowed airway is associated with wheezes (a high-pitched musical sound similar to a squeak).

The nurse is preparing to ambulate a postoperative client after cardiac surgery. The nurse plans to do which to enable the client to best tolerate the ambulation? 1. Provide the client with a walker. 2. Remove the telemetry equipment. 3. Encourage the client to cough and deep breathe. 4. Premedicate the client with an analgesic before ambulating.

correct answer 4. Premedicate the client with an analgesic before ambulating. Rationale: The nurse should encourage regular use of pain medication for the first 48 to 72 hours after cardiac surgery because analgesia will promote rest, decrease myocardial oxygen consumption caused by pain, and allow better participation in activities such as coughing, deep breathing, and ambulation.

The nurse is caring for a client with a diagnosis of myocardial infarction (MI) and is assisting the client in completing the diet menu. Which beverage does the nurse instruct the client to select from the menu? 1. Tea 2. Cola 3. Coffee 4.Lemonade

correct answer 4.Lemonade Rationale: A client with a diagnosis of MI should not consume caffeinated beverages. Caffeinated products can produce a vasoconstrictive effect leading to further cardiac ischemia. Coffee, tea, and cola all contain caffeine and need to be avoided in the client with MI.

The nurse is working with a client who has been diagnosed with Prinzmetal's (variant) angina. The nurse plans to reinforce which information about this type of angina when teaching the client? 1. Prinzmetal's angina is effectively managed by beta-blocking agents. 2. Prinzmetal's angina improves with a low-sodium, high-potassium diet. 3. Prinzmetal's angina has the same risk factors as stable and unstable angina. 4.Prinzmetal's angina is generally treated with calcium channel blocking agents.

correct answer 4.Prinzmetal's angina is generally treated with calcium channel blocking agents. Rationale: Prinzmetal's angina results from spasm of the coronary arteries and is generally treated with calcium channel blocking agents. The risk factors are unknown, and this type of angina is relatively unresponsive to nitrates. Beta blockers are contraindicated because they may actually worsen the spasm. Diet therapy is not specifically indicated, although a healthy diet consuming foods low in fat and sodium is advocated in cardiac disease.

A primary health care provider prescribes digoxin, 0.125 mg by mouth (PO) daily, for a client with heart failure. The medication label states 0.125 mg per tablet. How many tablet(s) will the nurse administer to the client? Fill in the blank.

correct answer 1 tablet(s) Rationale: Use the formula for calculating the appropriate medication dosage. In this question, it is not necessary to perform a conversion. desired x tablet(s)= tablet(s) per dose ___________ Available 0.125 ml x 1 tablet= 1 tablet ___________ 0.125 minutes

The nurse has completed counseling about smoking cessation with a client with coronary artery disease (CAD). The nurse determines that the client has understood the material best if the client makes which statement? 1. "A smoker has twice the risk of having a heart attack as a nonsmoker." 2. "I may try just cutting down first, because the damage has already been done." 3. "I don't think I want to quit because none of the effects are reversible anyway." 4. "I'm never going to start again because I can cut my risk of cardiovascular disease to zero within a year."

correct answer 1. "A smoker has twice the risk of having a heart attack as a nonsmoker." Rationale: Cigarette smokers have twice the risk of having a myocardial infarction as a nonsmoker and have two to four times the risk of having sudden cardiac death. Smoking cessation will reduce its damaging effects on the cardiovascular system; however, its cessation will not cut the risk to zero in 1 year.

The nurse provides medication instructions to an older hypertensive client who is taking 20 mg of lisinopril orally daily. The nurse evaluates the need for further teaching when the client makes which statement? 1. "I can skip a dose once a week." 2"I need to change my position slowly." 3"I take the pill after breakfast each day." 4"If I get a bad headache, I should call my health care provider immediately."

correct answer 1. "I can skip a dose once a week." Rationale: Lisinopril is an antihypertensive angiotensin-converting enzyme inhibitor. The usual dosage range is 20 to 40 mg per day. Adverse effects include headache, dizziness, fatigue, orthostatic hypotension, tachycardia, and angioedema. Specific client teaching points include taking one pill a day, not stopping the medication without consulting the health care provider (HCP), and monitoring for side effects and adverse reactions. The client should notify the HCP if side effects occur.

The nurse is providing discharge teaching for a post-myocardial infarction (MI) client who will be taking 1 baby aspirin a day. The nurse determines that the client understands the use of this medication if the client makes which statement? 1. "I will take this medication every day." 2. "I will take this medication every other day." 3. "I will take this medication until I feel better." 4. "I will take this medication only when I have pain."

correct answer 1. "I will take this medication every day." Rationale: A single daily dose of 1 baby aspirin (low-dose aspirin) may be a component of the standard treatment regimen for the client after an MI. Aspirin helps prevent clotting and may prevent a thrombosis that could cause a second MI. If the client cannot tolerate aspirin, then another antiplatelet medication may be prescribed. The other three options are unacceptable because the benefit comes in taking the medication on a daily basis.

The nurse determines that a client with coronary artery disease (CAD) understands disease management if the client makes which statement? 1. "I will walk for one-half hour daily." 2. "As long as I exercise I can eat anything I wish." 3. "My weight has nothing to do with this disease." 4. "It doesn't matter if my father had high cholesterol."

correct answer 1. "I will walk for one-half hour daily." Rationale: Lack of physical exercise contributes to the development of CAD and engaging in a regular program of exercise helps retard progression of atherosclerosis by lowering cholesterol levels and developing collateral circulation to heart tissue. Options 2 and 3 are incorrect because obesity and a diet high in fat can contribute to CAD. Option 4 is incorrect because genetic factors also contribute to CAD.

The nurse determines that a client with coronary artery disease (CAD) understands disease management if the client makes which statement? 1. "I will walk for one-half hour daily." 2. "As long as I exercise I can eat anything I wish." 3. "My weight has nothing to do with this disease." 4. "It doesn't matter if my father had high cholesterol."

correct answer 1. "I will walk for one-half hour daily." Rationale: Lack of physical exercise contributes to the development of CAD, and engaging in a regular program of exercise helps retard progression of atherosclerosis by lowering cholesterol levels and developing collateral circulation to heart tissue. Options 2 and 3 are incorrect because obesity and a diet high in fat can contribute to CAD. Option 4 is incorrect because genetic factors also contribute to CAD.

A client with a history of angina pectoris complains of substernal chest pain. The nurse checks the client's blood pressure and administers nitroglycerin 0.4 mg sublingually. Five minutes later, the client is still experiencing chest pain. If the blood pressure is still stable, the nurse would take which action next? 1. Administer another nitroglycerin tablet. 2Apply 1 to 3 L/minute of oxygen via nasal cannula. 3Call for a 12-lead electrocardiogram (ECG) to be performed. 4Wait an additional 5 minutes, then give a second nitroglycerin tablet.

correct answer 1. Administer another nitroglycerin tablet. Rationale: In the hospitalized client, nitroglycerin tablets usually are prescribed 1 every 5 minutes as needed (PRN) for chest pain up to a total dose of 3 tablets. The nurse in this question should administer the second tablet. The client with known angina pectoris should have low-flow oxygen at a rate of 1 to 3 L/minute via nasal cannula if pain is not relieved. A 12-lead ECG would be done if prescribed by standing protocol or by individual primary health care provider prescription.

The licensed practical nurse (LPN) in a medical unit is caring for a client with heart failure. The client suddenly develops extreme dyspnea, tachycardia, and lung crackles, and the nurse suspects pulmonary edema. The LPN immediately notifies the registered nurse (RN) and expects which interventions to be prescribed? Select all that apply. 1. Administering oxygen 2. Inserting a Foley catheter 3. Administering furosemide 4. Administering morphine sulfate intravenously 5. Transporting the client to the coronary care unit 6. Placing the client in a low-Fowler's side-lying position

correct answer 1. Administering oxygen 2. Inserting a Foley catheter 3. Administering furosemide 4. Administering morphine sulfate intravenously Rationale: Pulmonary edema is a life-threatening event that can result from severe heart failure. During pulmonary edema the left ventricle fails to eject sufficient blood, and pressure increases in the lungs because of the accumulated blood. Oxygen is always prescribed, and the client is placed in a high-Fowler's position to ease the work of breathing. Furosemide, a rapid-acting diuretic, will eliminate accumulated fluid. A Foley catheter is inserted to accurately measure output. Intravenously administered morphine sulfate reduces venous return (preload), decreases anxiety, and reduces the work of breathing. Transporting the client to the coronary care unit is not a priority intervention. In fact, this may not be necessary at all if the client's response to treatment is successful.

The nurse is attending an in-service education session on the therapeutic use of calcium-channel blockers. The instructor of the session determines that teaching has been effective when the nurse correctly identifies that these medications are used for which disorders? Select all that apply. 1. Angina 2. Glaucoma 3. Hypertension 4. Dysrhythmias 5. Acute kidney injury 6. Glomerulonephritis

correct answer 1. Angina 3. Hypertension 4. Dysrhythmias Rationale: Calcium-channel blockers are medications that prevent calcium ions from entering cells. These agents have their greatest effects on the heart and blood vessels. They are used widely to treat hypertension, angina pectoris, and cardiac dysrhythmias. They are not used to treat glaucoma, acute kidney injury, or glomerulonephritis.

The nurse on a telemetry unit checks a client's chart and notes that the potassium level is 6.3 mEq/L. Based on this laboratory result, which signs/symptoms would the nurse anticipate? Select all that apply. 1. Anxiety 2. Bradycardia 3. Hypertension 4. Hyperglycemia 5. Electrocardiogram (ECG) changes

correct answer 1. Anxiety 5. Electrocardiogram (ECG) changes Rationale: Potassium is important in nerve conduction and contraction of all muscle types. Increased potassium, or hyperkalemia, can cause muscle weakness in all muscles, which can progress to paralysis. In the heart, this can result in cardiac arrest and death. Cardiovascular changes are the most significant and most dangerous problems associated with hyperkalemia. Hyperkalemia is associated with ECG changes such as tall, peaked T waves, absent or flat P waves, and wide QRS complexes. Hyperkalemia is also associated with anxiety and irritability. Bradycardia and hypoglycemia are associated with hypokalemia. Hypertension is not associated with either hyperkalemia or hypokalemia.

A client with a diagnosis of myocardial infarction has a new activity prescription allowing the client to have bathroom privileges. As the client stands and begins to walk, the client begins to complain of chest pain. The nurse would take which action? 1. Assist the client to get back into bed. 2. Report the chest pain episode to the health care provider. 3. Tell the client to stand still, and take the client's blood pressure. 4. Give a nitroglycerin tablet, and assist the client to the bathroom.

correct answer 1. Assist the client to get back into bed. Rationale: The client is assisted back to bed to put the client at rest. The nurse can then measure vital signs and administer nitroglycerin that is prescribed for as-needed (PRN) use. The nurse should then report the chest pain episode to the health care provider. The nurse should not continue to assist the client into the bathroom because it places the client in danger because of continued myocardial oxygen demands.

A low-sodium diet has been prescribed for a client with hypertension. Which food selected from the menu by the client indicates an understanding of this diet? 1. Baked turkey 2. Tomato soup 3. Boiled shrimp 4. Chicken gumbo

correct answer 1. Baked turkey Rationale: Regular soup (1 cup) contains 900 mg of sodium. Fresh shellfish (1 oz) contains 50 mg of sodium. Poultry (1 oz) contains 25 mg of sodium.

The nurse is collecting data from a client with hypertension being treated with diuretic therapy. The nurse would monitor the client for hypokalemia if the client is receiving which diuretic? 1. Bumetanide 2. Triamterene 3. Amiloride HCl 4. Spironolactone

correct answer 1. Bumetanide Rationale: Bumetanide is a loop diuretic, which places this client at risk for hypokalemia. The nurse assesses this client carefully for signs of hypokalemia, monitors serum potassium levels, and encourages intake of potassium sources in the diet. Spironolactone, triamterene, and amiloride HCl are potassium-retaining diuretics.

The nurse is caring for a client who had a total knee replacement. Postoperatively, the nurse monitors for which highest priority assessment? 1. Calf pain 2. Heel breakdown 3. Bladder distention 4. Extremity shortening

correct answer 1. Calf pain Rationale: The highest priority assessment that the nurse needs to monitor in this client is the presence of calf pain. Deep vein thrombosis is a potentially serious complication of lower extremity surgery. Calf pain is a sign of this complication. Although bladder distention may occur postoperatively, this assessment is incorrect because it is not specific to the information in the question. Extremity lengthening or shortening may occur as a result of knee replacement but is not the highest priority. Additionally, heel breakdown is not the highest priority.

The nurse is caring for a client who had a total knee replacement. Postoperatively, the nurse monitors for which highest priority assessment? 1. Calf pain 2. Heel breakdown 3. Bladder distention 4.Extremity shortening

correct answer 1. Calf pain Rationale: The highest priority assessment that the nurse needs to monitor in this client is the presence of calf pain. Deep vein thrombosis is a potentially serious complication of lower extremity surgery. Calf pain is a sign of this complication. Although bladder distention may occur postoperatively, this assessment is incorrect because it is not specific to the information in the question. Extremity lengthening or shortening may occur as a result of knee replacement but is not the highest priority. Additionally, heel breakdown is not the highest priority.

The nurse is reinforcing dietary instructions to a client with heart failure (HF). The nurse determines that the client understands the instructions if the client states that which food item will be avoided? 1. Catsup 2. Sherbet 3. Cooked cereal 4. Leafy green vegetables

correct answer 1. Catsup Rationale: Catsup is high in sodium. Leafy green vegetables, cooked cereal, and sherbet all are low in sodium. Clients with heart failure should monitor sodium intake.

The nurse is reinforcing dietary instructions to a client with heart failure (HF). The nurse determines that the client understands the instructions if the client states that which food item will be avoided? 1. Catsup 2Sherbet 3Cooked cereal 4Leafy green vegetables

correct answer 1. Catsup Rationale: Catsup is high in sodium. Leafy green vegetables, cooked cereal, and sherbet all are low in sodium. Clients with heart failure should monitor sodium intake.

The nurse is caring for a cardiac client who has recently displayed this monitored rhythm. Which actions by the nurse are most appropriate? Refer to figure. Select all that apply. 1. Check all telemetry leads. 2. Check blood pressure (BP). 3. Review cardiac laboratory markers. 4. Obtain stat electrocardiogram (ECG). 5. Review client's history and physical data. 6. Assess client's level of consciousness (LOC).

correct answer 1. Check all telemetry leads. 2. Check blood pressure (BP). 4. Obtain stat electrocardiogram (ECG). 6. Assess client's level of consciousness (LOC). Rationale: When this rhythm of atrial fibrillation is noted on a client's monitor, the nurse needs to check all telemetry leads to confirm the rhythm, check the client's BP and level of consciousness, and then obtain a stat ECG. Reviewing the client's cardiac laboratory markers and history and physical can be done later.

A client has received instructions about an upcoming cardiac catheterization. The nurse determines that the client has the best understanding of the procedure if the client knows to report which symptoms? 1. Chest pain 2. Urge to cough 3. Warm, flushed feeling 4. Pressure at the insertion site

correct answer 1. Chest pain Rationale: The client is taught before cardiac catheterization to immediately report chest pain or any unusual sensations. The client is taught that a warm, flushed feeling may accompany dye injection, occasional palpitations may occur, and the urge to cough may occur as the catheter tip touches the cardiac muscle. The client may be asked to cough or breathe deeply from time to time during the procedure. Because a local anesthetic is used, the client should feel pressure, but not pain, at the insertion site.

The nurse is caring for a client with a new onset of atrial fibrillation. Which prescribed treatments would the nurse expect? Select all that apply. 1. Digoxin 2. Warfarin 3. Amiodarone 4. Defibrillation 5. Electrical cardioversion

correct answer 1. Digoxin 2. Warfarin 5. Electrical cardioversion Rationale: The three goals of treatment for atrial fibrillation are ventricular rate control, prevention of embolic stroke, and restoration and maintenance of normal sinus rhythm. Digoxin is used for ventricular rate control. Warfarin is used to decrease the risk of embolic stroke. Electrical cardioversion is used to restore normal sinus rhythm. Amiodarone is used to treat ventricular tachycardia or ventricular fibrillation. Defibrillation is the treatment of choice for ventricular fibrillation.

The nurse would anticipate the use of which medications in the treatment of the client with heart failure? Select all that apply. 1. Diuretics 2. Anticoagulants 3. Anticholinergics 4. Cardiac glycosides 5. Phosphodiesterase (PDE) inhibitors 6. Angiotensin-converting enzyme (ACE) inhibitors

correct answer 1. Diuretics 4. Cardiac glycosides 5. Phosphodiesterase (PDE) inhibitors 6. Angiotensin-converting enzyme (ACE) inhibitors Rationale: Medications recommended for treatment of heart failure include diuretics, cardiac glycosides such as digoxin, PDE inhibitors, and ACE inhibitors. Clients in heart failure do not need anticoagulants or anticholinergics.

The nurse is caring for a client with a permanent pacemaker. The nurse knows that which three primary problems can occur when cardiac pacemakers malfunction? Select all that apply. 1. Failure to sense 2. Failure to demand 3. Failure to capture 4. Failure to pace or fire 5. Failure to transmit data 6. Failure to join pacer cells

correct answer 1. Failure to sense 3. Failure to capture 4. Failure to pace or fire Rationale: The three primary problems associated with pacemaker malfunction include failure to sense when the pacemaker does not sense the client's own cardiac rhythm, failure to capture when an electrical impulse or spike is generated but no depolarization is noted, and failure to pace or fire when the pacemaker fails to initiate an electrical stimulus. Failure to demand, failure to transmit data, and failure to join pacer cells are not problems associated with the functioning of a pacemaker.

The nurse working the 3:00 to 11:00 pm shift notes that a client with coronary artery disease (CAD) has a prescription for serum lipid levels to be drawn in the morning. The nurse places the client on which dietary preparation to ensure accurate test results? 1. Fasting for 12 hours 2. Added snack at bedtime 3. Nothing by mouth (NPO) for 24 hours 4. Early meal tray delivered before the laboratory test

correct answer 1. Fasting for 12 hours Rationale: To obtain an accurate cholesterol level, a client must fast 12 hours before the tests. Options 2 and 4 interfere with accurate test results, and option 3 represents an unnecessary time period.

A client with hypertension has been prescribed a low-sodium diet. The nurse reinforcing instructions about foods that are allowed would include which foods in a list provided to the client? Select all that apply. 1. Fresh tomato 2. Tomato soup 3. Boiled shrimp 4. Instant oatmeal 5. Summer squash

correct answer 1. Fresh tomato 5. Summer squash Rationale: Foods that are lower in sodium are fruits and vegetables, such as fresh tomato and summer squash, because they do not contain physiological saline. Highly processed or refined foods, such as prepared soups and cereal, are higher in sodium unless they are noted specifically to be "low sodium." Saltwater fish and shellfish (shrimp) are high in sodium.

The nurse is caring for a client with coronary artery disease, and a topical nitrate is prescribed for the client. Why is acetaminophen usually prescribed to be taken before the administration of the topical nitrate? 1. Headache is a common side effect of nitrates. 2. Fever usually accompanies coronary artery disease. 3. Acetaminophen potentiates the therapeutic effects of nitrates. 4. Acetaminophen does not interfere with platelet action as acetylsalicylic acid does.

correct answer 1. Headache is a common side effect of nitrates. Rationale: Headache occurs as a side effect of nitrates. Acetaminophen may be given before nitrates to prevent headaches or to minimize the discomfort from the headaches. Option 2 is incorrect. Options 3 and 4 are unrelated to the data in the question.

The nurse is preparing to administer digoxin, 0.125 mg orally, to a client with heart failure. Which vital sign is most important for the nurse to check before administering the medication? 1. Heart rate 2. Temperature 3. Respirations 4. Blood pressure

correct answer 1. Heart rate Rationale: Digoxin is a cardiac glycoside that is used to treat heart failure and acts by increasing the force of myocardial contraction. Because bradycardia may be a clinical sign of toxicity, the nurse counts the apical heart rate for 1 full minute before administering the medication. If the pulse rate is less than 60 beats/minute in an adult client, the nurse would withhold the medication and report the pulse rate to the registered nurse, who would then contact the primary health care provider.

The nurse is told during shift report that a client is having occasional ventricular dysrhythmias. The nurse reviews the client's laboratory results, recalling that which electrolyte imbalance could be responsible for this development? 1. Hypokalemia 2. Hypernatremia 3. Hypochloremia 4. Hypercalcemia

correct answer 1. Hypokalemia Rationale: The nurse assesses the client's serum laboratory results for hypokalemia. The client may experience ventricular dysrhythmias in the presence of hypokalemia because this electrolyte imbalance increases the electrical instability of the heart. The electrolyte imbalances mentioned in the other options do not have this effect.

Which cardiovascular sign would the nurse expect to note in a client with a diagnosis of hypocalcemia? 1. Hypotension 2. Increased heart rate 3. Bounding peripheral pulses 4. Shortened QT interval on electrocardiogram (ECG)

correct answer 1. Hypotension Rationale: Cardiovascular manifestations that occur with hypocalcemia include decreased heart rate, diminished peripheral pulses, and hypotension. On the ECG, the nurse should note a prolonged ST segment and a prolonged QT interval.

The nurse is monitoring the status of the postoperative client after abdominal surgery earlier in the day. Which signs or symptoms noted by the nurse would indicate an evolving complication associated with hypovolemia? Select all that apply. 1. Increasing restlessness 2. Capillary refill of 3 seconds in all extremities 3Hypoactive bowel sounds in all four quadrants 4. White blood cell (WBC) count 9,500 mm3 (9.5 × 109/L) 5Blood pressure of 104/66 mm Hg with a pulse of 106 beats per minute

correct answer 1. Increasing restlessness 5Blood pressure of 104/66 mm Hg with a pulse of 106 beats per minute Rationale: Hypovolemia is decreased circulating blood volume. In a postoperative client, this often is associated with inadequate fluid replacement or hemorrhage. Increasing restlessness is a sign that requires continuous and close monitoring because it could forecast a complication such as shock. A low and dropping BP with an increased pulse rate could be early compensation for a decrease in circulating blood volume. The WBC count is normal at 5000 to 10,000 mm3 (5 to 10 × 109/L). Hearing hypoactive bowel sounds in all four quadrants is a normal occurrence, as is a capillary refill of 3 seconds in all extremities.

A comatose client received therapeutic hypothermia after a cardiac arrest. The nurse anticipates which primary complications associated with this treatment? Select all that apply. 1. Infection 2. Bleeding 3. Hypoglycemia 4. Pressure ulcers 5. Renal insufficiency 6. Metabolic and electrolyte disturbances

correct answer 1. Infection 2. Bleeding 6. Metabolic and electrolyte disturbanc Rationale: Induced hypothermia is an option for comatose adult clients with return of spontaneous circulation (ROSC) after an in-hospital cardiac arrest of any initial rhythm, or after out-of-hospital cardiac arrest with an initial rhythm of pulseless electrical activity (PEA) or asystole. Potential complications for clients receiving therapeutic hypothermia after a cardiac arrest include infection, bleeding, and metabolic and electrolyte imbalances. Hyperglycemia and not hypoglycemia may occur. Pressure ulcers are not anticipated and renal insufficiency is not an expected complication.

A client with infective endocarditis is at risk for heart failure. The nurse monitors the client for which signs and symptoms of heart failure? 1. Lung crackles, peripheral edema, and weight gain 2. Confusion, decreasing level of consciousness, and aphasia 3. Respiratory distress, chest pain, and the use of accessory muscles 4. Flank pain with radiation to the groin, accompanied by hematuria

correct answer 1. Lung crackles, peripheral edema, and weight gain Rationale: The client with infective endocarditis may experience both left- and right-sided heart failure, and thus the nurse monitors the client for both pulmonary and peripheral symptoms such as lung crackles, peripheral edema, and weight gain. Options 2 and 4 relate to disorders of the brain and kidney, respectively. Option 3 contains symptoms that occur with pulmonary embolism, which is not related to the subject of the question.

The health care provider is discharging a client with a diagnosis of primary hypertension. Which health maintenance instructions would the nurse reinforce in the discharge teaching plan? Select all that apply. 1. Monitor the blood pressure at home. 2. Restrict sodium intake as prescribed. 3. Take a calcium supplement to lower blood pressure. 4. Eye examinations with an ophthalmoscope should be routine. 5. Follow-up appointments for blood pressure checks are important.

correct answer 1. Monitor the blood pressure at home. 2. Restrict sodium intake as prescribed. 4. Eye examinations with an ophthalmoscope should be routine. 5. Follow-up appointments for blood pressure checks are important. Rationale: Primary hypertension is a condition that increases the risk of cardiovascular disease and renal disease. Home self-measure blood pressure monitoring should be done as prescribed to monitor the client's response to prescribed treatment. Follow-up appointments for blood pressure checks are also important to monitor the client's response to treatment. Sodium should be restricted as prescribed to prevent elevations in the blood pressure. Regular ophthalmoscopic examinations are needed to detect retinal changes seen in hypertensive clients. The use of calcium supplements to lower blood pressure is not known and therefore is not recommended.

A hospitalized client with heart disease who is taking digoxin has a digoxin level prescribed. The level is elevated above normal. Based on this finding the nurse plans to notify the registered nurse and primary health care provider (PHCP) and anticipates which additional interventions will be prescribed? Select all that apply. 1. Monitor the potassium level. 2. Place the client on cardiac monitor. 3. Prescribe a repeat digoxin level for the next day. 4. Prescribe an additional dose of digoxin for today. 5Monitor the blood urea nitrogen (BUN) and creatinine.

correct answer 1. Monitor the potassium level. 2. Place the client on cardiac monitor. 5Monitor the blood urea nitrogen (BUN) and creatinine. Rationale: Digoxin is a cardiac glycoside that is used as a second-line medication to treat heart failure. It affects the electrical and mechanical actions of the heart and can reach toxic levels easily. An elevated digoxin level is digoxin toxicity. Digoxin toxicity with cardiac dysrhythmias is enhanced by hypokalemia so the electrolytes, especially potassium, should be monitored. Digoxin is excreted through the kidneys so renal function should be determined by the BUN and creatinine levels. The client should be placed on a cardiac monitor so the client can be assessed for dysrhythmias. A repeat digoxin level is not needed because digoxin has a long half-life, and the result would not be noticeably different so soon. An additional dose should not be given due to the elevated digoxin level.

The nurse is preparing for a health fair about tobacco use and the development of coronary heart disease. Which information would the nurse include? Select all that apply. 1. Nicotine decreases oxygen to the heart. 2. Hypnosis may be helpful to stop smoking. 3. Avoid exposure to environmental tobacco smoke. 4. Cigars or pipes are healthier than cigarette smoking. 5. Tobacco smoking increases a female's level of estrogen.

correct answer 1. Nicotine decreases oxygen to the heart. 2. Hypnosis may be helpful to stop smoking. 3. Avoid exposure to environmental tobacco smoke. Rationale: Tobacco use is a major risk factor for the development of coronary heart disease. Nicotine vasoconstricts the arteries, causing a decrease in myocardial oxygen supply and an increase in demand. To successfully quit smoking, it is necessary to combine multiple strategies. Hypnosis is a complementary/alternative therapy smoking cessation strategy. Exposure to environmental tobacco (secondhand) smoke does increase the risk for the development of coronary heart disease. Cigar or pipe smokers have an increased risk for the development of coronary heart disease similar to environmental tobacco smoke. Tobacco smoking decreases estrogen levels in premenopausal women, increasing their risk of coronary heart disease.

A licensed practical nurse (LPN) is assisting in the care of a client receiving a continuous intravenous (IV) infusion of heparin sodium for deep vein thrombosis (DVT). The LPN notes that the result of a newly drawn activated partial thromboplastin time (aPTT) level is 90 seconds. The client's baseline before the initiation of therapy was 30 seconds. The LPN would take which action? 1. Notify the RN about the value immediately. 2. Ask the client about worsening pain from the DVT. 3. Check to see if additional heparin is available on the unit. 4. Leave the report for the registered nurse (RN) to review later in the day.

correct answer 1. Notify the RN about the value immediately. Rationale: The normal aPTT varies between 20 and 36 seconds, depending on the type of activator used in testing. When a client receives intravenous heparin, the range of the aPTT is ordered by the primary health care provider but is greater than the normal range. Heparin treatment for DVT often involves a protocol to follow determined by the results of aPTT. If the aPTT is within the desired level, the rate is maintained and an aPTT is not ordered again until the next morning. The LPN should report the findings immediately to the RN, who will take further action to follow up on the elevated value. Checking for pain from the DVT, checking for additional heparin, and delaying reporting the aPTT are not appropriate actions.

The primary health care provider is discharging a client with a diagnosis of chronic heart failure. Which health maintenance instructions would the nurse reinforce in the discharge teaching plan? Select all that apply. 1. Obtain annual influenza vaccination. 2. Restrict fluid intake to 1000 mL per day. 3. Avoid adding salt to foods or in cooking. 4. Report a weight gain of 3 or more pounds in a week. 5. Take an extra dose of prescribed diuretic for swollen ankles.

correct answer 1. Obtain annual influenza vaccination. 3. Avoid adding salt to foods or in cooking. 4. Report a weight gain of 3 or more pounds in a week. Rationale: Heart failure is a chronic illness and requires lifelong treatment with a focus on health maintenance. Annual influenza vaccination is recommended to prevent the flu. Avoiding dietary sodium will decrease intravascular volume. A weight gain of 3 or more pounds in a week most likely indicates fluid retention and needs to be reported to the health care provider. Fluid restrictions are not commonly prescribed for chronic heart failure, although the client may be advised to monitor intake. The client should not change the dose of any medicine without talking with the primary health care provider.

A client with left-sided heart failure has been admitted to the hospital. The nurse is reviewing the medical record and notes which signs and symptoms? Select all that apply. 1. Orthopnea 2. Weight gain 3. Sleep apnea 4. Pitting edema 5. Pink frothy sputum

correct answer 1. Orthopnea 3. Sleep apnea 5. Pink frothy sputum Rationale: Left-sided signs and symptoms include fatigue, dyspnea, wheezing, orthopnea, sleep apnea, pulmonary edema (pink, frothy sputum), pallor, and clammy skin. Right-sided heart failure signs and symptoms include fatigue, edema in sacrum, legs, feet, ankles, hepatomegaly, abdominal distention as a result of ascites, weight gain, and dyspnea.

A client who weighs 50 kg has arrived in the emergency department complaining of severe chest pain. The telemetry monitor shows an evolving anterior myocardial infarction (MI). The nurse anticipates that the primary health care provider will initially prescribe which treatments? Select all that apply. 1. Oxygen 2 L, per nasal cannula 2. Chewable aspirin 324 mg, oral 3. Nitroglycerin 0.4 mg, sublingual 4. Heparin 5000 units, subcutaneously 5. Morphine sulfate 4 mg, intravenously

correct answer 1. Oxygen 2 L, per nasal cannula 2. Chewable aspirin 324 mg, oral 3. Nitroglycerin 0.4 mg, sublingual 5. Morphine sulfate 4 mg, intravenously Rationale: Oxygen will be provided to help provide oxygenation to the heart muscle. Nitroglycerin will be administered in an effort to cause vasodilation. Morphine sulfate will be given to decrease the client's pain and to improve myocardial oxygenation. Chewable aspirin is administered to inhibit platelet aggregation and the development of thrombus. Heparin is not indicated initially. It may be used later on in conjunction with other medications for certain types of myocardial infarctions or unstable angina.

The nurse is evaluating the effects of care for the client with deep vein thrombosis. Which limb observations should the nurse note as indicating the least success in meeting the outcome criteria for this problem? 1. Pedal edema that is 3+ 2. Slight residual calf tenderness 3. Skin warm, equal temperature both legs 4. Calf girth ¼ inch larger than unaffected limb

correct answer 1. Pedal edema that is 3+ Rationale: Symptoms of deep vein thrombosis include leg warmth, redness, edema, tenderness, and enlarged calf. If the problem is not resolved, or is minimally resolved, these symptoms will remain. Option 3 indicates full resolution of the problem, whereas options 2 and 4 indicate partial resolution. Option 1 is the correct option because it indicates the least degree of symptom reversal.

The nurse is caring for a client with left-sided heart failure. Which clinical signs are most important for the nurse to communicate to the primary health care provider? Select all that apply. 1. Pink-tinged frothy sputum 2. Increase in respiratory rate 3. Ankle and lower leg swelling 4. Paroxysmal nocturnal dyspnea 5. Auscultation of crackles throughout the lungs

correct answer 1. Pink-tinged frothy sputum 2. Increase in respiratory rate 5. Auscultation of crackles throughout the lungs Rationale: Left-sided heart failure can lead to pulmonary edema or acute decompensated heart failure (ADHF). Pink-tinged frothy sputum, an increase in respiratory rate (tachypnea), and auscultation of crackles throughout the lungs are signs of pulmonary edema caused by excess fluid accumulation in the alveoli. These signs need to be communicated to the health care provider because pulmonary edema requires immediate emergency treatment. Ankle and lower leg swelling and paroxysmal nocturnal dyspnea are clinical signs of chronic heart failure.

A client is receiving a continuous heparin infusion for venous thromboembolism treatment. Which laboratory monitoring would the nurse plan to check during a continuous heparin infusion? Select all that apply. 1. Platelets 2. Prothrombin time (PT) 3. Recombinant factor VIIa 4. International normalized ratio (INR) 5Activated partial thromboplastin time (aPTT)

correct answer 1. Platelets 5Activated partial thromboplastin time (aPTT) Rationale: A continuous heparin infusion requires monitoring the platelet count for heparin-induced thrombocytopenia and activated partial thromboplastin time to monitor blood clotting time. Warfarin is monitored using the international normalized ratio (INR). The INR is a standardized system of reporting prothrombin time (PT). Recombinant factor VIIa is used to reverse the anticoagulant effect of fondaparinux.

A client with complaints of mild shortness of breath and weakness comes to the medical clinic. The nurse reviews the client's chart and immediately contacts the primary health care provider about which life-threatening finding? Refer to chart. History and Physical Laboratory/Diagnostic Results Chronic kidney disease Potassium: 7 mEq/L Diabetes mellitus Blood glucose: 152 mg/dL Heart failure Chest x-ray: slight pleural effusion Medications Lasix 40 mg orally daily Insulin corrective scale as prescribed Oxygen 2 to 4 liters as needed 1. Potassium level 2. Chest x-ray result 3. Blood glucose level 4. History of diabetes mellitus

correct answer 1. Potassium level Rationale: A potassium level of 7 mEq/L or greater can cause heart blocks, asystole, and ventricular fibrillation. Immediate intervention is needed to quickly reduce that lethal potassium level. The blood glucose and chest x-ray results are within normal limits for this client. The blood glucose is elevated, but the client has diabetes mellitus.

The nurse is preparing to administer furosemide to a client with a diagnosis of heart failure. Which is the most important laboratory test result for the nurse to review before administering this medication? 1. Potassium level 2. Creatinine level 3. Cholesterol level 4. Blood urea nitrogen (BUN)

correct answer 1. Potassium level Rationale: Furosemide is a loop diuretic. The medication causes a decrease in the client's electrolytes, especially potassium, sodium, and chloride. Administering furosemide to a client with a low potassium level could precipitate ventricular dysrhythmias. The options of BUN and creatinine reflect renal function. The cholesterol level is unrelated to the administration of this medication.

The nurse is assigned to assist in caring for a client who has had surgery and has pneumatic sequential compression devices (SCDs) in place. The client asks about these devices. The nurse instructs the client that SCDs are used for which purpose? 1. Promoting venous return to the heart 2. Preventing edema in the lower extremities 3. Improving oxygenation to the lower extremities 4. Decreasing the size of any thrombus that formed during surgery

correct answer 1. Promoting venous return to the heart Rationale: Pneumatic sequential compression devices (SCDs) are placed on the client's lower extremities during and after surgery to prevent venous thrombotic embolic complications. The SCDs promote venous return to the heart decreasing the risk of deep venous thrombosis and pulmonary embolism. SCDs are not used to prevent edema or improve oxygenation to the lower extremities. SCDs are not used to treat an existing thrombus.

A hospitalized client with angina continues to have chest pain after the initial administration of a sublingual nitroglycerin tablet. The nurse would take which action? 1. Provide a second sublingual dose in 5 minutes. 2. Continue dosing at 10-minute intervals for 1 hour. 3. Instruct the client to swallow the next tablet whole. 4. Use distraction techniques such as deep breathing and imagery.

correct answer 1. Provide a second sublingual dose in 5 minutes. Rationale: To terminate an acute anginal attack, sublingual nitroglycerin should be administered as soon as the pain begins. Administration should not be delayed until the pain has become severe. In the hospitalized client, if pain is not relieved in 5 minutes, the client should take another dose sublingually, and then a third tablet is taken 5 minutes later. Anginal pain that does not respond to nitroglycerin may indicate myocardial infarction. If the client is having a myocardial infarction, distraction techniques such as deep breathing and imagery will not relieve the pain.

The primary health care provider has prescribed morphine sulfate intravenous push for a client with pulmonary edema. Which therapeutic effects would the nurse expect in this client? Select all that apply. 1. Relief of anxiety 2. Decreased respiratory rate 3. Reduction of oxygen consumption 4. Prevention of cardiac dysrhythmias 5. Improvement in efficacy of breathing

correct answer 1. Relief of anxiety 3. Reduction of oxygen consumption 5. Improvement in efficacy of breathing Rationale: Pulmonary edema is a condition caused by excess fluid in the lungs. It is a complication of cardiac disease such as heart failure, pneumonia, and other conditions. The fluid collects in the numerous air sacs in the lungs, making it difficult to breathe. Pulmonary edema requires pharmacological treatment. Morphine sulfate administrated by intravenous push is used in clients with pulmonary edema because it relieves anxiety and the client's fear of impending doom. This in turn improves the efficacy of breathing and decreases oxygen demand. Decreasing the respiratory rate and preventing cardiac dysrhythmias are not therapeutic effects of morphine.

A 24-year-old man seeks medical attention for complaints of claudication in the arch of the foot. The nurse also notes superficial thrombophlebitis of the lower leg. Next, the nurse would check the client's medical history for which item? 1. Smoking history 2. Recent exposure to allergens 3. History of recent insect bites 4. Familial tendency toward peripheral vascular disease

correct answer 1. Smoking history Rationale: The mixture of arterial and venous manifestations (claudication and phlebitis, respectively) in the young male client suggests thromboangiitis obliterans (Buerger's disease). This is a relatively uncommon disorder characterized by inflammation and thrombosis of smaller arteries and veins. This disorder is typically found in young men who smoke. The cause is unknown but is suspected to have an autoimmune component.

A client has just been treated with cardioversion. The nurse should check which measure first? 1. Status of airway 2. Blood pressure 3. Oxygen flow rate 4. Level of consciousness

correct answer 1. Status of airway Rationale: Nursing responsibilities after cardioversion include maintenance of a patent airway, oxygen administration, assessment of vital signs and level of consciousness, and dysrhythmia detection. Airway, however, is always the highest priority.

The client's B-type natriuretic peptide (BNP) level is 691 pg/mL. Which intervention would the nurse institute when providing care for the client? 1. Take daily weights and monitor trends. 2. Encourage fluids to improve hydration. 3. Elevate the legs above the level of the heart. 4. Position supine with the head of the bed at 30 degrees.

correct answer 1. Take daily weights and monitor trends. Rationale: BNP levels greater than 500 pg/mL indicate that heart failure is probable. Nursing measures are geared toward decreasing intravascular volume, decreasing preload, and decreasing afterload. Fluids increase intravascular volume, and elevating the legs above the level of the heart and positioning supine with the head of the bed at 30 degrees increase preload.

The nurse is assisting in caring for a client in the telemetry unit who is receiving an intravenous infusion of 1000 mL 5% dextrose with 40 mEq of potassium chloride. Which occurrence observed on the cardiac monitor indicates the presence of hyperkalemia? 1. Tall, peaked T waves 2. ST-segment depression 3. Prolonged P-R interval 4. Widening of the QRS complex

correct answer 1. Tall, peaked T waves Rationale: In the client with hypokalemia, the nurse would note ST-segment depression on a cardiac monitor. The client may also exhibit a flat T wave. Options 1, 3, and 4 are cardiac findings noted in the client with hyperkalemia.

The nurse is assisting in caring for a client in the telemetry unit who is receiving an intravenous infusion of 1000 mL 5% dextrose with 40 mEq of potassium chloride. Which occurrence observed on the cardiac monitor indicates the presence of hyperkalemia 1. Tall, peaked T waves 2. ST segment depressions 3. Shortened P-R intervals 4. Shortening of the QRS complex

correct answer 1. Tall, peaked T waves Rationale: The symptoms of hyperkalemia relate to its effect on the myocardial muscle. These include changes noted on the ECG such as tall, peaked T waves, prolonged P-R interval, widening of the QRS complex, shortening of the Q-T interval, and disappearance of the P wave. Other cardiac symptoms include ventricular dysrhythmias that may lead to cardiac arrest. ST-segment depression is noted in hypokalemia.

A client with no history of heart disease has experienced an acute myocardial infarction and been given thrombolytic therapy with tissue plasminogen activator (tPA). The nurse interprets that the client is likely experiencing a complication of this therapy if which occurs? 1. Tarry stools 2. Orange-colored urine 3. Nausea and vomiting 4. Decreased urine output

correct answer 1. Tarry stools Rationale: Thrombolytic agents are used to dissolve existing thrombi, and the nurse must monitor the client for obvious or occult signs of bleeding. This includes assessment for obvious bleeding within the gastrointestinal (GI) tract, urinary system, and skin. It also includes testing secretions for occult blood. Option 1 is the only option that indicates the presence of blood Orange-colored urine is associated with administration of the antibiotic rifampin. Orange-colored urine, nausea and vomiting, and decreased urine output are not complications associated with administration of tPA.

The nurse is assisting in the care of a client diagnosed with rheumatic heart disease. The nurse would reinforce instructions to the client to notify the dentist before dental procedures for which reason? 1. The client requires prophylactic antibiotics before treatment. 2. The dentist should use a low-speed drill to avoid dysrhythmias. 3. The dentist should use a lidocaine solution without epinephrine. 4. The client is at risk for episodes of heart failure triggered by stressful events.

correct answer 1. The client requires prophylactic antibiotics before treatment. Rationale: The client with a history of rheumatic fever is at risk for developing infective endocarditis. The client should tell all health care providers and dentists about this problem so that prophylactic antibiotic therapy can be given before any procedure that is invasive or carries a risk of bleeding. Low-speed drills, epinephrine, and stressful events are unrelated to rheumatic heart disease.

The primary health care provider (PHCP) is going to perform carotid massage on a client with rapid rate atrial fibrillation. Which interventions would the nurse anticipate? Select all that apply. 1. The client should be placed on a cardiac monitor. 2. The PHCP massages the carotid artery for a full minute. 3. The head should be turned toward the side to be massaged. 4. Rhythm strips should be obtained before, during, and after the procedure. 5. Monitor the vital signs, cardiac rhythm, and level of consciousness after the procedure.

correct answer 1. The client should be placed on a cardiac monitor. 4. Rhythm strips should be obtained before, during, and after the procedure. 5. Monitor the vital signs, cardiac rhythm, and level of consciousness after the procedure. Rationale: Carotid sinus massage is one maneuver used for vagal stimulation to decrease a rapid heart rate and possibly terminate a tachydysrhythmia. This eliminates option 3. The PHCP or cardiologist will massage only one carotid artery for a few seconds to determine whether a change in cardiac rhythm occurs. This eliminates option 2. The client needs to be on a cardiac monitor throughout the procedure, and rhythm strips should be obtained before, during, and after the procedure. Continue to monitor the client's cardiac rhythm as well as vital signs and level of consciousness.

The nurse plans care understanding that the primary reason clients experience vasodilation in septic shock is because: 1. There is a release of endotoxins from bacteria. 2There is heart failure with diminished cardiac output .3There is blood or fluid loss and the body compensates by dilating the blood vessels .4There is an obstruction of blood flow, and the body compensates by dilating the blood vessels.

correct answer 1. There is a release of endotoxins from bacteria. Rationale: A massive infection leads to sepsis as a result of a endotoxins being released, which causes vasodilation, pooling of blood, and capillary permeability. The remaining options do not provide the reason for vasodilation in septic shock.

Acetylsalicylic acid is prescribed for a client before a percutaneous transluminal coronary angioplasty (PTCA). When the nurse takes the acetylsalicylic acid to the client, the client asks the nurse about its purpose. What is the purpose of the acetylsalicylic acid? 1. To prevent the formation of clots 2. To relieve pain at the injection site 3. To prevent a fever after the procedure 4. To prevent inflammation of the injection site

correct answer 1. To prevent the formation of clots Rationale: Before PTCA, the client is usually given an anticoagulant, commonly acetylsalicylic acid, to help reduce the risk of occlusion of the artery during the procedure. Pain, fever, and inflammation are unrelated to the purpose of administering acetylsalicylic acid to this client.

Which electrocardiogram changes would the nurse note on the cardiac monitor with a client whose potassium (K+) level is 2.7 mEq/L (2.7 mmol/L)? 1. U waves 2. Flat P waves 3. Elevated T waves 4.Prolonged PR interval

correct answer 1. U waves Rationale: A serum potassium level less than 3.5 mEq/L (3.5 mmol/L) is indicative of hypokalemia. Potassium deficit is the most common electrolyte imbalance and is potentially life-threatening. Cardiac changes with hypokalemia may include peaked P waves, flattened T waves, depressed ST segment, and the presence of U waves.

A client with hyperlipidemia is seen in the clinic for a follow-up visit. Which dietary modifications would the nurse include to lower the risk of coronary heart disease? Select all that apply. 1. Use liquid vegetable oil. 2. Increase intake of fruits. 3. Choose whole grain foods. 4. Remove skin from poultry. 5. Select whole milk products.

correct answer 1. Use liquid vegetable oil. 2. Increase intake of fruits. 3. Choose whole grain foods. 4. Remove skin from poultry. Rationale: Hyperlipidemia is a modifiable risk factor for the development of coronary heart disease. Reducing the amount of dietary saturated fat and cholesterol helps lower the risk for coronary heart disease. Dietary modifications such as using liquid vegetable oil, eating fresh fruits and whole grain foods, and removing the skin from poultry will lower dietary fat. The client should also use low-fat or fat-free (skim) milk in place of whole milk products to lower dietary fat.

The nurse is caring for a postpartum client with a diagnosis of thrombophlebitis. The client suddenly complains of chest pain and dyspnea. The nurse would initially check which item? 1. Vital signs 2. Fundal height 3. Presence of calf pain 4. Level of consciousness (LOC)

correct answer 1. Vital signs Rationale: Pulmonary embolism is a complication of thrombophlebitis. Changes in the vital signs are one of the first things to occur with pulmonary embolism, because pulmonary blood flow is compromised. Fundal height is unrelated to the information in the question. Calf pain is an indicator of thrombophlebitis. Level of consciousness may change as the condition worsens; worsening would indicate hypoxia.

A client with angina pectoris has just been started on medication therapy with nitroglycerin. In planning care for this client, the nurse would place priority on measuring which data? 1. Vital signs 2. Serum glucose 3. Intake and output 4. Therapeutic serum drug levels

correct answer 1. Vital signs Rationale: The nurse would place priority on measuring vital signs, especially the blood pressure, because of the vasodilator action of the medication. Drug levels are not measured for nitroglycerin, and the medication does not affect serum glucose level. Intake and output may be measured as part of the general plan of care for the client with heart disease, but it is not directly related to administration of this medication.

A client is being discharged from the hospital on warfarin for venous thromboembolism (VTE) prevention. Which instructions would the nurse reinforce in the client's teaching plan? Select all that apply. 1. Wear a Medic Alert bracelet. 2. Check urine and stool for blood. 3. Notify your dentist before an appointment. 4. Take a double dose to make up for a missed dose. 5. You do not have to take the medication at the same time each day.

correct answer 1. Wear a Medic Alert bracelet. 2. Check urine and stool for blood. 3. Notify your dentist before an appointment. Rationale: Warfarin is an anticoagulant, and bleeding is the major complication. Teaching for the client includes wearing a Medic Alert bracelet to notify health care providers that an anticoagulant is being taken. Blood in the urine and stool may indicate the dose is too high. Certain dental procedures require temporarily stopping an anticoagulant because of the risk of bleeding. The prescribed dosage must be taken at the same time each day to maintain therapeutic effect. A double dose to make up for a missed dose is not done because this will increase the risk of bleeding.

The nurse is assisting a client admitted to the hospital with pulmonary edema to prepare for discharge. The nurse would reinforce with the client the importance of complying with which measure to prevent a recurrence. 1. Weigh self every morning before breakfast. 2. Sleep with the head elevated on only one pillow. 3. Take additional digoxin if respiratory distress occurs. 4. Adjust diuretic dose based on severity of peripheral edema.

correct answer 1. Weigh self every morning before breakfast. Rationale: A long-range approach to the prevention of pulmonary edema is to minimize any pulmonary congestion. The client should weigh himself or herself daily as a means of determining fluid balance and possible overload. The client should sleep with the head elevated as high as needed to prevent pulmonary congestion during sleep. The client should not self-adjust any medication dosages.

The nurse at an outpatient cardiology clinic is reviewing the medical history of a 48-year-old client during a routine exam. The client is complaining of the inability to maintain an erection and asks the nurse what could be causing it. Which information would the nurse include as possible contributing factors to his erectile dysfunction? Select all that apply. 1. Weight 245 lb 2Total cholesterol 223 mg/dL 3. Serum creatinine 1.86 mg/dL 4. Blood pressure 117/68 mm Hg 5. Thyroid stimulating hormone (TSH) 1.54 mIU/L

correct answer 1. Weight 245 lb 2Total cholesterol 223 mg/dL 3. Serum creatinine 1.86 mg/d Rationale: Erectile dysfunction affects an individual between 40 to 70 years old. Risk factors for erectile dysfunction mimic risk factors for cardiovascular disease. Obesity, hyperlipidemia, chronic kidney disease, hypertension, hypothyroidism, diabetes mellitus, smoking, alcohol abuse, and a lack of exercise all increase the risk for erectile dysfunction. A weight of 245 lb indicates the client may be overweight, a total cholesterol level of 223 mg/dL is elevated, and a serum creatinine of 1.86 mg/dL may suggest chronic kidney disease. Blood pressure and TSH are within normal limits.

A client with a diagnosis of myocardial infarction has a new activity prescription allowing the client to have bathroom privileges. As the client stands and begins to walk, the client begins to complain of chest pain. The nurse would take which action? 1.Assist the client to get back into bed. 2. Report the chest pain episode to the health care provider. 3. Tell the client to stand still, and take the client's blood pressure. 4. Give a nitroglycerin tablet, and assist the client to the bathroom.

correct answer 1.Assist the client to get back into bed. Rationale: The client is assisted back to bed to put the client at rest. The nurse can then measure vital signs and administer nitroglycerin that is prescribed for as-needed (PRN) use. The nurse should then report the chest pain episode to the health care provider. The nurse should not continue to assist the client into the bathroom because it places the client in danger because of continued myocardial oxygen demands.

A primary health care provider prescribes digoxin 0.5 mg orally daily for a client with heart failure. The medication label states "0.25 mg per tablet." How many tablet(s) will the nurse administer to the client? Fill in the blank. _____ tablet(s)

correct answer 2 tablet(s) Rationale: Follow the formula for the calculation of the correct dose. desired x tablet+ tablet(s) per dose ___________ available 0.5mg x 1 tablet = 2 tablets __________ 0.25

The nurse is teaching the client with angina pectoris about disease management and lifestyle changes that are necessary in order to control disease progression. Which statements by the client indicate a need for further teaching? Select all that apply. 1. "I will avoid using table salt with meals." 2"I am going to switch to electronic cigarettes." 3"It is best to exercise once a week for an hour." 4"I will take nitroglycerin whenever chest discomfort begins." 5"I will use muscle relaxation to cope with stressful situations."

correct answer 2"I am going to switch to electronic cigarettes." 3"It is best to exercise once a week for an hour." Rationale: Exercise is most effective when done at least 3 times a week for 20 to 30 minutes to reach a target heart rate. Other healthy habits include limiting salt and fat in the diet and using stress management techniques. The client also should be taught to take nitroglycerin before any activity that causes pain, and to take the medication at the first sign of chest discomfort. Electronic cigarettes contain nicotine, which causes vasoconstriction.

The nurse is teaching the client with angina pectoris about disease management and lifestyle changes that are necessary in order to control disease progression. Which statements by the client indicate a need for further teaching? Select all that apply. 1. "I will avoid using table salt with meals." 2. "I am going to switch to electronic cigarettes." 3. "It is best to exercise once a week for an hour." 4. "I will take nitroglycerin whenever chest discomfort begins." 5. "I will use muscle relaxation to cope with stressful situations."

correct answer 2. "I am going to switch to electronic cigarettes." 3. "It is best to exercise once a week for an hour." Rationale: Exercise is most effective when done at least 3 times a week for 20 to 30 minutes to reach a target heart rate. Other healthy habits include limiting salt and fat in the diet and using stress management techniques. The client also should be taught to take nitroglycerin before any activity that causes pain, and to take the medication at the first sign of chest discomfort. Electronic cigarettes contain nicotine, which causes vasoconstriction.

The nurse has reinforced instructions to a client receiving enalapril maleate. Which statement by the client indicates a need for further teaching? 1. "I need to rise slowly from a lying to sitting position." 2. "I need to notify the primary health care provider if nausea occurs." 3. "I need to notify the primary health care provider if a sore throat occurs." 4. "I know that several weeks of therapy may be required for the full therapeutic effect."

correct answer 2. "I need to notify the primary health care provider if nausea occurs." Rationale: If nausea occurs, it is not necessary to notify the primary health care provider. The client should be instructed to consume noncaffeinated carbonated beverages, unsalted crackers, or dry toast to alleviate the nausea. To reduce the hypotensive effect of this medication, the client is instructed to rise slowly from a lying to a sitting position and to permit the legs to dangle from the bed momentarily before standing. The client should report signs of a sore throat or fever to the health care provider because these may indicate infection. The client should be notified that several weeks may be needed for the full therapeutic effect of blood pressure reduction. The client should also be instructed not to skip doses or discontinue the medication because severe rebound hypertension can occur.

The nurse reinforces instructions to a client at risk for thrombophlebitis regarding measures to minimize its occurrence. Which statement by the client indicates an understanding of this information? 1. "I need to avoid pregnancy by taking oral contraceptives." 2. "I should avoid sitting in one position for long periods of time." 3. "I can finally stop wearing these support stockings that you gave me." 4. "I will be sure to maintain my fluid intake to at least four glasses daily."

correct answer 2. "I should avoid sitting in one position for long periods of time. Rationale: Avoidance of sitting or standing for a prolonged period of time is one of the measures for the prevention of venous stasis and thrombophlebitis. Taking oral contraceptives causes hypercoagulability that could result in thrombophlebitis. Support stockings are used to promote venous return, to maintain normal coagulability, and to prevent injury to the endothelial wall. Adequate hydration is maintained to prevent hypercoagulability, and four glasses daily are an inadequate amount of fluid.

The nurse reinforces instructions to a client at risk for thrombophlebitis regarding measures to minimize its occurrence. Which statement by the client indicates an understanding of this information? 1. "I need to avoid pregnancy by taking oral contraceptives." 2. "I should avoid sitting in one position for long periods of time." 3. "I can finally stop wearing these support stockings that you gave me." 4. "I will be sure to maintain my fluid intake to at least four glasses daily."

correct answer 2. "I should avoid sitting in one position for long periods of time." Rationale: Avoidance of sitting or standing for a prolonged period of time is one of the measures for the prevention of venous stasis and thrombophlebitis. Taking oral contraceptives causes hypercoagulability that could result in thrombophlebitis. Support stockings are used to promote venous return, to maintain normal coagulability, and to prevent injury to the endothelial wall. Adequate hydration is maintained to prevent hypercoagulability, and four glasses daily are an inadequate amount of fluid.

The nurse determines that a client with coronary artery disease (CAD) needs further teaching about disease management if the client makes which statement? 1. "I will watch my weight gain." 2. "I will avoid walking for exercise." 3. "I will monitor my cholesterol intake." 4. "I will follow a low-fat, low-salt diet."

correct answer 2. "I will avoid walking for exercise." Rationale: Lack of physical exercise contributes to the development of coronary artery disease, and engaging in a regular program of exercise helps retard progression of atherosclerosis by lowering cholesterol levels and developing collateral circulation to heart tissue. Walking should be encouraged for 30 minutes a day. Watching weight gain; monitoring cholesterol; and following a low-fat, low-salt diet are accurate statements.

The nurse has reinforced instructions to the client with Raynaud's disease about self-management of the disease process. The nurse determines that the client needs further teaching if the client makes which statement? 1. "Smoking cessation is very important." 2. "Moving to a warmer climate should help." 3. "Sources of caffeine should be eliminated from the diet." 4. "Taking nifedipine as prescribed will decrease vessel spasm."

correct answer 2. "Moving to a warmer climate should help Rationale: Raynaud's disease responds favorably to the elimination of nicotine and caffeine. Medications such as calcium channel blockers may inhibit vessel spasm and prevent symptoms. Avoiding exposure to cold through a variety of means is very important. However, moving to a warmer climate may not necessarily be beneficial because the symptoms could still occur with the use of air conditioning and during periods of cooler weather.

The nurse is collecting data on a client with a diagnosis of angina pectoris who takes nitroglycerin for chest pain. During the admission, the client reports chest pain. The nurse immediately asks the client which question? 1. "Are you having any nausea?" 2. "Where is the pain located?" 3. "Are you allergic to any medications?" 4. "Do you have your nitroglycerin with you?"

correct answer 2. "Where is the pain located?" Rationale: If a client complains of chest pain, the initial assessment question is to ask the client about the pain intensity, precipitating factors, location, radiation, and quality. Although options 1, 3, and 4 may be components of the assessment, these would not be the initial assessment questions in this situation.

A client who has undergone femoropopliteal bypass grafting says to the nurse, "I hope I don't have any more problems that could make me lose my leg. I'm so afraid that I'll have gone through this for nothing." Which is an appropriate nursing response? 1. "There is nothing to worry about." 2. "You are concerned about losing your leg?" 3. "There are many people with the same problem, and they are doing just fine." 4. "You have the best health care provider in the city, and your health care provider will not let anything happen to you."

correct answer 2. "You are concerned about losing your leg?" Rationale: The appropriate response is the one that uses the therapeutic technique of restatement. Option 2 restates the client's concern and provides an opportunity for the client to further discuss the concern. Options 1, 3, and 4 are inappropriate because they provide false reassurance and do not address the client's concern.

A hospitalized client with coronary artery disease complains of substernal chest pain. After checking the client's heart rate and blood pressure, the nurse administers nitroglycerin, 0.4 mg sublingually. After 5 minutes, the client states, "My chest still hurts." If the vital signs have remained stable, which action would the nurse perform? 1. Apply 10 L of oxygen via nasal cannula. 2. Administer another nitroglycerin tablet. 3. Call the resuscitation team immediately. 4. Administer a second nitroglycerin tablet in 10 minutes.

correct answer 2. Administer another nitroglycerin tablet. Rationale: Nitroglycerin tablets usually are prescribed 1 every 5 minutes as needed (PRN) for chest pain, for a total dose of 3 tablets. Waiting 10 minutes is inappropriate if the client is having chest pain. Oxygen at 10 L is an unsafe dose. There is no need to call the resuscitation team at this time.

The nurse notes this rhythm on a client's cardiac monitor. The nurse next reports that the client is experiencing which heart rhythm? Refer to figure. 1. Normal sinus 2. Atrial fibrillation 3. Sinus bradycardia 4. Ventricular fibrillation

correct answer 2. Atrial fibrillation Rationale: Atrial fibrillation is characterized by no distinct P waves and an irregular ventricular response. In sinus bradycardia and normal sinus rhythm, there will be clear distinct P waves and a regular ventricular rhythm. In ventricular fibrillation. there are no clear P waves or QRS complexes.

The nurse is reinforcing instructions to a client with angina pectoris about measures to reduce recurrence of chest pain. The nurse would stress to the client the importance of taking which measure? 1. Saving all chores for the end of the day 2. Avoiding exposure to either very hot or very cold weather 3. Eating large meals to reduce the work of the gastrointestinal tract 4. Keeping items stored above shoulder level to encourage exercise

correct answer 2. Avoiding exposure to either very hot or very cold weather Rationale: The client should avoid extreme hot or cold temperatures to avoid placing undue stress on the cardiovascular system. The client should space activities throughout the day rather than save them for the end of the day when the client is more fatigued. The client should eat smaller meals so less blood flow is diverted for the work of digestion. Exercise is important, but the client should keep most items stored at heart level to prevent straining and increased intrathoracic pressure, which can decrease cardiac output.

The nurse is reinforcing instructions to a client with angina pectoris about measures to reduce recurrence of chest pain. The nurse would stress to the client the importance of taking which measure? 1. Saving all chores for the end of the day 2. Avoiding exposure to either very hot or very cold weather 3. Eating large meals to reduce the work of the gastrointestinal tract 4.Keeping items stored above shoulder level to encourage exercise

correct answer 2. Avoiding exposure to either very hot or very cold weather Rationale: The client should avoid extreme hot or cold temperatures to avoid placing undue stress on the cardiovascular system. The client should space activities throughout the day rather than save them for the end of the day when the client is more fatigued. The client should eat smaller meals so less blood flow is diverted for the work of digestion. Exercise is important, but the client should keep most items stored at heart level to prevent straining and increased intrathoracic pressure, which can decrease cardiac output.

The nurse is planning a dietary menu for a client with heart failure who is being treated with digoxin and furosemide. Which would be the best dinner choice from the daily menu? 1. Beef ravioli, spinach soufflé, and Italian bread 2. Baked fish, mashed potatoes, and carrot-raisin salad 3. Roasted chicken breast, brown rice, and stewed tomatoes 4. Beef vegetable soup, macaroni and cheese, and a dinner roll

correct answer 2. Baked fish, mashed potatoes, and carrot-raisin salad Rationale: Furosemide depletes potassium, and a client on digoxin and furosemide needs to maintain normal potassium levels and moderate salt intake. Hypokalemia may make the client more susceptible to digoxin toxicity. The recommended daily intake for potassium is 2000 mg. Option 4 is not the best choice because beef vegetable soup contains a high amount of sodium and a minimal amount of potassium. Macaroni and cheese is also high in sodium and contains no potassium. Option 1 is not the best choice because beef ravioli is high in sodium and contains no potassium. Spinach soufflé is a good source of potassium but also contains sodium. Option 3 is not the best choice because roasted chicken breast, brown rice, and stewed tomatoes contain a minimal amount of potassium. Option 2 is the best choice because all three foods are high in potassium and low in sodium.

A client with myocardial infarction (MI) has been transferred from the coronary care unit (CCU) to the general medical unit with cardiac monitoring via telemetry. The nurse assisting in caring for the client expects to note which type of activity prescribed? 1. Strict bed rest for 24 hours 2. Bathroom privileges and self-care activities 3. Unrestricted activities because the client is monitored 4. Unsupervised hallway ambulation with distances less than 200 feet

correct answer 2. Bathroom privileges and self-care activities Rationale: Upon transfer from the coronary care unit, the client is allowed self-care activities and bathroom privileges. Supervised ambulation in the hall for brief distances is encouraged, with distances gradually increased (50, 100, 200 feet).

A client has a history of left-sided heart failure. The nurse would look for the presence of which finding to determine whether the problem is currently active? 1. Presence of ascites 2. Bilateral lung crackles 3. Jugular vein distention 4. Pedal edema bilaterally

correct answer 2. Bilateral lung crackles Rationale: The client with heart failure may present with different symptoms depending on whether the right or the left side of the heart is failing. Breath sounds are an accurate indicator of left-sided heart function. Peripheral edema, jugular vein distention, and ascites can be present as a result of insufficiency of the pumping action of the right side of the heart.

A client with myocardial infarction is a candidate for alteplase therapy. The nurse assisting in the care of this client is aware that it will be necessary to monitor for which adverse effect of this therapy? 1. Infection 2. Bleeding 3. Allergic reaction 4. Muscle weakness

correct answer 2. Bleeding Rationale: Alteplase is a thrombolytic medication, which means that it breaks down or dissolves clots. Because of its action, the principal adverse effect is bleeding. Local or systemic infection could occur with poor aseptic technique during medication administration, but it is rare. Allergic reaction is not a frequent response. Muscle weakness is not an adverse effect of this medication.

A client is wearing a continuous cardiac monitor, which begins to alarm at the nurse's station. The nurse sees no electrocardiographic complexes on the screen. The nurse would do which action first? 1. Call a code blue. 2. Check the client status and lead placement. 3. Call the primary health care provider (PHCP). 4. Press the recorder button on the ECG console.

correct answer 2. Check the client status and lead placement. Rationale: Sudden loss of electrocardiographic complexes indicates ventricular asystole or possibly electrode displacement. Checking of the client and equipment is the first action by the nurse.

A client with myocardial infarction suddenly becomes tachycardic, shows signs of air hunger, and begins coughing frothy, pink-tinged sputum. The nurse listens to breath sounds expecting to hear which breath sounds bilaterally? 1. Rhonchi 2. Crackles 3. Wheezes 4. Diminished breath sounds

correct answer 2. Crackles Rationale: Pulmonary edema is characterized by extreme breathlessness, dyspnea, air hunger, and production of frothy, pink-tinged sputum. Auscultation of the lungs reveals crackles. Wheezes, rhonchi, and diminished breath sounds are not associated with pulmonary edema.

A client is admitted with chest pain related to atrial fibrillation. Based on her blood glucose reading, metformin is prescribed for the client. As the nurse reviews the client's chart and prescriptions, which finding would require the nurse to verify the metformin prescription? Refer to chart. History and Physical Laboratory/ Diagnostic Results History of atrial fibrillation I Blood urea nitrogen (BUN): 22 mg/dL and migraine headaches I Blood urea nitrogen (BUN): 22 mg/dLSumatriptan I Creatinine: 2.2 mg/dL (Imitrex) 25 mg as needed orally Assessment:Blood pressure: 160/90 mm Hg, I Hemoglobin A1C: 8.9% pulse: 88 beats per minut States urinary difficulty. I Blood glucose: 240 mg/dL Medication Sumatriptan (Imitrex) 25 mg I as needed orally I Propranolol (Inderal) 10 mg, I 3 times daily, orally I Metformin (Glucophage) I 500 mg twice daily, orally 1. Blood pressure 2. Creatinine result 3. Hemoglobin A1c 4. Migraine headaches

correct answer 2. Creatinine result Rationale: Metformin is contraindicated with a creatinine level greater than 1.4 mg/dL. Although the blood pressure is elevated, the client is on a beta blocker. The hemoglobin A1c is elevated, as is the blood glucose level, suggesting the need for an antidiabetic medication. Migraine headaches do not affect the kidneys.

A client is admitted with an arterial ischemic leg ulcer. The nurse expects to note that this ulcer has which typical characteristic? 1. Dark, pink base 2. Deep and painful 3. Accompanied by very slight pain 4. Brown pigmentation of surrounding skin

correct answer 2. Deep and painful Rationale: Arterial leg ulcers tend to be deep and painful. The client usually has rest pain, and the ulcer site is painful. Surrounding skin has coloration consistent with peripheral arterial disease. Options 1, 3, and 4 are not characteristics of an arterial leg ulcer.

An emergency department client who complains of slightly improved but unrelieved chest pain for 2 days is reluctant to take a nitroglycerin sublingual tablet offered by the nurse. The client states, "I don't need that—my dad takes that for his heart. There's nothing wrong with my heart." Which description best describes the client's response? 1. Angry 2. Denial 3. Phobic 4. Obsessive-compulsive

correct answer 2. Denial Rationale: Denial is the most common reaction when a client has a myocardial infarction or anginal pain. No angry behavior was identified in the question. Phobias and obsessive-compulsive disorders are mental health diagnoses.

A client admitted to the hospital with a diagnosis of myocardial infarction (MI) tells the nurse that the pain likely resulted from the fried chicken sandwich that the client had for lunch. The nurse's response is based on which fact? 1. Most people love high-fat diets. 2. Denial is a common occurrence early after MI. 3. The client probably wants to belittle the opinion of the staff. 4. The client is not motivated to learn about heart disease at this time.

correct answer 2. Denial is a common occurrence early after MI. Rationale: An early initial coping response following MI is denial. The nurse uses this knowledge of this common response in planning care for the client. Option 1 is an opinion and not based on information in the question. There is no evidence in the question to support options 3 and 4.

A client admitted to the hospital with coronary artery (CAD) disease complains of dyspnea at rest. The nurse determines that which would be of most help to the client? 1. Providing a walker to aid in ambulation 2. Elevating the head of the bed to at least 45 degrees 3. Performing continuous monitoring of oxygen saturation 4. Placing an oxygen cannula at the bedside for use if needed

correct answer 2. Elevating the head of the bed to at least 45 degrees Rationale: The management of dyspnea generally is directed toward alleviation of the cause. Symptom relief may be achieved or at least aided by placing the client at rest with the head of the bed elevated. Supplemental oxygen may be used but placing equipment at the bedside is not directly helpful. Monitoring of oxygen saturation detects early complications but does not help the client. Likewise, placing an oxygen cannula at the bedside for use would not help the client.

The nurse is assisting in developing a plan of care for a client who will be returning to the nursing unit following a cardiac catheterization via the femoral approach. Which nursing intervention would be included in the postprocedure plan of care? 1. Place the client's bed in the Fowler's position. 2. Encourage the client to increase fluid intake. 3. Instruct the client to perform range-of-motion exercises of the extremities. 4. Hold regularly scheduled medications for 24 hours following the procedure.

correct answer 2. Encourage the client to increase fluid intake. Rationale: Immediately following a cardiac catheterization using the femoral approach, the client should not flex or hyperextend the affected leg. Placing the client in the Fowler's position increases the risk of hemorrhage. Fluids are encouraged to assist in removing the contrast medium from the body. Asking the client to move the toes is done to assess motion, which could be impaired if a hematoma or thrombus were developing. Flexion or hyperextension and range-of-motion exercises of the extremity are contraindicated. The regularly scheduled medications are needed to treat acute and chronic conditions.

A client is at risk for complications of heart failure. Which is the nurse's priority for early detection of the most likely cause of complications with this client? 1. Checking vital signs 2. Evaluating total body fluid 3. Reviewing serum electrolytes 4. Monitoring electrocardiogram

correct answer 2. Evaluating total body fluid Rationale: Fluid overload can cause complications for the client with heart failure. Therefore, the nurse evaluates the client's fluid balance to forestall activation of harmful compensatory mechanisms and deterioration of other organ systems that increasing total body fluid can cause. This is the nurse's priority because balancing the client's fluid status has the broadest range of potential benefits for the client including improving oxygenation. The vital signs, serum electrolytes, and electrocardiogram are important assessments yet remain secondary in importance to fluid status because they are items that are affected by fluid balance.

The nurse is collecting data on a client who was just admitted to the hospital with a diagnosis of coronary artery disease (CAD). The client reveals having been under a great deal of stress recently. Which would the nurse do next? 1. Ask whether the client wants to see a psychiatrist. 2. Explore with the client the sources of stress in life. 3. Reassure the client that everybody seems stressed these days. 4. Ask the client to write down a list of stressors to be evaluated at a later time.

correct answer 2. Explore with the client the sources of stress in life. Rationale: The nurse should encourage the client to explore and verbalize stressors. Later, the nurse can teach the client strategies for coping with stress such as the basic relaxation techniques of deep breathing, progressive muscle relaxation, and visualization. Asking whether the client wants to see a psychiatrist could be construed as excessive or insulting and puts the client's feelings on hold. Reassuring the client that everybody seems stressed these days ignores the client's concerns. Asking the client to write down a list of stressors places further data collection of this area on hold.

A client who has undergone a cardiac catheterization using the right femoral approach is returned to the nursing unit. Thirty minutes later the client complains of numbness and tingling of the right foot. The pedal pulse is weak, and the foot is pale. The nurse notifies the registered nurse because these symptoms are consistent with which problem? 1. Right sciatic nerve damage 2. Femoral artery thrombus or hematoma 3. Local allergic reaction to the contrast dye 4. Early massive infection at the catheter insertion site

correct answer 2. Femoral artery thrombus or hematoma Rationale: Adverse changes such as numbness and tingling, coolness, pallor, cyanosis, or sudden loss of peripheral pulses indicate serious circulatory impairment and are reported to the registered nurse immediately, who then contacts the primary health care provider. Allergic reaction to the dye is a systemic problem, not a local one. The data in the question are not consistent with sciatic pain. Infection does not become apparent this quickly.

The nurse is assisting a client who will wear a Holter monitor for continuous cardiac monitoring over the next 24 hours. The nurse takes which action to assist the client? 1. Shaves the front of the client's chest 2. Gives the client a device holder to wear around the waist 3. Teaches the client to rest as much as possible during the next 24 hours 4. Tells the client to cover the monitor in plastic wrap before taking a bath

correct answer 2. Gives the client a device holder to wear around the waist Rationale: The nurse applies electrocardiographic (ECG) monitoring leads to the chest in the usual fashion and gives the client a sling or holder to carry the transistor-sized monitor that is worn around the chest or waist. The nurse would remind the client to maintain a normal schedule and to keep a diary of all activity and symptoms. The client should avoid activities that could interfere with the ECG recorder such as using heavy machinery, electric shavers, hair dryers, or bathing or showering. Therefore, options 1, 3, and 4 are incorrect.

A client with known coronary artery disease (CAD) begins to experience chest pain while getting out of bed. The nurse would take which action? 1. Get a prescription for pain medication. 2. Have the client stop and lie back down in bed. 3. Report the complaint to the health care provider. 4. Have the client continue to get out of bed and into a chair.

correct answer 2. Have the client stop and lie back down in bed. Rationale: The pain associated with coronary artery disease is called angina pectoris, and it occurs because of myocardial tissue ischemia from insufficient blood flow to the heart. The nurse should first have the client stop the activity and lie back down to decrease the workload and oxygen demand on the heart. Getting a prescription for pain medicine and reporting the complaint to the health care provider can be done after ensuring that the client is resting. The pain medication that is likely to be prescribed is nitroglycerin, which is a coronary vasodilator. Having the client continue to get out of bed and into a chair is contraindicated and will worsen the pain and possibly lead to myocardial infarction.

A client with atrial fibrillation who is receiving maintenance therapy of warfarin sodium has a prothrombin time (PT) of 35 seconds and an international normalized ratio (INR) of 3.5. On the basis of these laboratory values, the nurse anticipates which prescription? 1. Adding a dose of heparin sodium 2. Holding the next dose of warfarin 3. Increasing the next dose of warfarin 4. Administering the next dose of warfarin

correct answer 2. Holding the next dose of warfarin Rationale: The normal PT is 11 seconds to 12.5 seconds (conventional therapy and SI units). The normal INR is 0.81 to 1.2 (conventional therapy and SI units); 2 to 3 for standard warfarin therapy, which is used for the treatment of atrial fibrillation, and 3 to 4.5 for high-dose warfarin therapy, which is used for clients with mechanical heart valves. A therapeutic PT level is 1.5 to 2 times higher than the normal level. Because the values of 35 seconds and 3.5 are high, the nurse should anticipate that the client would not receive further doses at this time. Therefore, the prescriptions noted in the remaining options are incorrect.

The nurse is monitoring a client with an abdominal aortic aneurysm (AAA). Which finding is probably unrelated to the AAA? 1. Pulsatile abdominal mass 2. Hyperactive bowel sounds in the area 3. Systolic bruit over the area of the mass 4. Subjective sensation of "heart beating" in the abdomen

correct answer 2. Hyperactive bowel sounds in the area Rationale: Not all clients with abdominal aortic aneurysm exhibit symptoms. Those who do may describe a feeling of the "heart beating" in the abdomen when supine or being able to feel the mass throbbing. A pulsatile mass may be palpated in the middle and upper abdomen. A systolic bruit may be auscultated over the mass. Hyperactive bowel sounds are not specifically related to an abdominal aortic aneurysm.

A client has just completed an information session about measures to minimize the progression of coronary artery disease (CAD). Which statement indicates an initial understanding of lifestyle alterations? 1. I should take daily medication for life. 2. I should eat a diet that is low in fat and cholesterol. 3. I should continue to smoke to keep the metabolic rate high. 4. I should begin to exercise if diet is not sufficient to achieve weight loss.

correct answer 2. I should eat a diet that is low in fat and cholesterol Rationale: A diet that is low in fat and cholesterol helps slow the progression of CAD. This must be accompanied by regular exercise and cessation of smoking. If these measures are effective, the client may not need daily medication.

A client has just completed an information session about measures to minimize the progression of coronary artery disease (CAD). Which statement indicates an initial understanding of lifestyle alterations? 1. I should take daily medication for life. 2. I should eat a diet that is low in fat and cholesterol. 3. I should continue to smoke to keep the metabolic rate high. 4. I should begin to exercise if diet is not sufficient to achieve weight loss.

correct answer 2. I should eat a diet that is low in fat and cholesterol. Rationale: A diet that is low in fat and cholesterol helps slow the progression of CAD. This must be accompanied by regular exercise and cessation of smoking. If these measures are effective, the client may not need daily medication.

A client who has heart failure receives an additional dose of bumetanide as prescribed 4 hours after the daily dose. The nurse assesses the client 15 minutes after administering the medication and reminds the client to save all urine in the bathroom. Sixty minutes later the nurse finds the client on the floor, unresponsive, and bleeding from a laceration. Which issues support the client's malpractice claim? Select all that apply. 1. Failure to replace body fluids 2. Increased risk of hypotension 3. Lack of follow-up nursing actions 4. Increased need to protect the client 5. Failure to teach the client adequately 6. Excessive bumetanide administration

correct answer 2. Increased risk of hypotension 3. Lack of follow-up nursing actions 4. Increased need to protect the client 5. Failure to teach the client adequately Rationale: To prove malpractice against the nurse, the plaintiff must prove that the nurse owed a duty to the client; that the nurse breached the duty; and that as a result, harm was caused to person or property. The client has an increased risk of hypotension (option 2) because hypotension is a common adverse effect of bumetanide. This is the second dose within 4 hours, and the client has heart failure. The client can prove that the nurse did not protect him by failing to provide adequate teaching and perform correct and timely nursing interventions after administering the bumetanide. After the first 15-minute check, the nurse should continue increased client monitoring to ensure client compliance with safety measures. Replacing fluid volume is not the issue; furthermore, the goal of therapy is to reduce total body fluid. No data indicate that the dose of bumetanide, a loop diuretic, was excessive. However, because this medication can cause hypotension, especially after a repeat dose, the nurse should instruct the client to remain in bed and provide him with a urinal. It may be difficult for the client to prove that the second dose of bumetanide caused the injury.

The nurse is teaching a hospitalized client who has had aortoiliac bypass grafting about measures to improve circulation. The nurse would tell the client to do which? 1. Bend the leg at the hip. 2. Keep the ankles uncrossed. 3. Place two pillows under the knees. 4. Use the knee gatch on the bed controls.

correct answer 2. Keep the ankles uncrossed. Rationale: A graft can become clotted from any form of pressure that results in impaired blood flow through the graft. Positions and movements to be avoided include bending at the hip or knee, crossing the knees or ankles, or the use of a knee gatch or pillows under the knees.

The nurse is assisting with caring for the client immediately after insertion of a permanent demand pacemaker via the right subclavian vein. The nurse prevents dislodgement of the pacing catheter by implementing which intervention? 1. Limiting movement and abduction of the left arm 2. Limiting movement and abduction of the right arm 3. Assisting the client to get out of bed and ambulate with a walker 4. Having the physical therapist do active range of motion to the right arm

correct answer 2. Limiting movement and abduction of the right arm Rationale: In the first several hours after insertion of either a permanent or temporary pacemaker, the most common complication is pacing electrode dislodgment. The nurse helps prevent this complication by limiting the client's activities.

The nurse is preparing to provide a therapeutic environment for a client who recently had a myocardial infarction (MI). Which are characteristics of a therapeutic environment? 1. No stimulus, no stress 2. Low stimulus, low stress 3. High stimulus, low stress 4. Moderate stimulus, low stress

correct answer 2. Low stimulus, low stress Rationale: An environment that is low stimulus and low stress is needed to decrease anxiety and metabolic demands for the client after MI. Nursing care is directed at promoting rest and assisting with activities of daily living. Option 1 cannot be provided, and options 3 and 4 are too high in stimulus to be therapeutic.

The nurse is checking the neurovascular status of a client who returned to the surgical nursing unit 4 hours ago after undergoing an aortoiliac bypass graft. The affected leg is warm, and the nurse notes redness and edema. The pedal pulse is palpable and unchanged from admission. Based on this data, the nurse would make which determination about the client's neurovascular status? 1. Moderately impaired, and the surgeon should be called 2. Normal, caused by increased blood flow through the leg 3. Slightly deteriorating and should be monitored for another hour 4. Adequate from an arterial approach, but venous complications are arising

correct answer 2. Normal, caused by increased blood flow through the leg Rationale: An expected outcome of surgery is warmth, redness, and edema in the surgical extremity caused by increased blood flow. Options 1, 3, and 4 are incorrect.

A client is admitted to the hospital with a diagnosis of pericarditis. The nurse reviews the client's record for which sign or symptom that differentiates pericarditis from other cardiopulmonary problems? 1. Anterior chest pain 2. Pericardial friction rub 3. Weakness and irritability 4. Chest pain that worsens on inspiration

correct answer 2. Pericardial friction rub Rationale: A pericardial friction rub is heard when there is inflammation of the pericardial sac during the inflammatory phase of pericarditis. Chest pain that worsens on inspiration is characteristic of both pericarditis and pleurisy. Anterior chest pain may be experienced with angina pectoris and myocardial infarction. Weakness and irritability are nonspecific complaints that could accompany a wide variety of disorders.

The nurse assisting in caring for a client hospitalized with acute pericarditis is monitoring the client for signs of cardiac tamponade. The nurse determines that which finding is unrelated to possible cardiac tamponade? 1. Distended jugular neck veins 2. Pulse rate of 58 beats per minute 3. Muffled and distant heart sounds 4. Systolic blood pressure of 110 mm Hg, dropping to 94 mm Hg on inspiration

correct answer 2. Pulse rate of 58 beats per minute Rationale: Assessment findings with cardiac tamponade include tachycardia, distant or muffled heart sounds, jugular vein distention, and a falling blood pressure, accompanied by pulsus paradoxus (a drop in inspiratory blood pressure by greater than 10 mm Hg). Bradycardia is the symptom that is unrelated.

A client in pulmonary edema has a prescription to receive morphine sulfate intravenously. The licensed practical nurse assisting in caring for the client determines that the client experienced an intended effect of the medication if which is noted? 1. Increased pulse rate 2. Relief of apprehension 3. Decreased urine output 4. Increased blood pressure

correct answer 2. Relief of apprehension Rationale: Morphine sulfate reduces anxiety and dyspnea in the client with pulmonary edema. It also promotes peripheral vasodilation and causes blood to pool in the periphery. It decreases pulmonary capillary pressures, which reduces fluid migration into the alveoli. The client receiving morphine sulfate is monitored for signs and symptoms of respiratory depression and extreme drops in blood pressure, especially when administered intravenously. Options 1, 3, and 4 are unrelated to the action of morphine sulfate.

A nurse is about to give a daily dose of digoxin and notes that a serum digoxin level drawn earlier in the day measured 2.7 ng/mL. The nurse would take which actions? Select all that apply. 1. Administer the daily dose of the medication. 2. Report the finding to the registered nurse. 3. Administer foods rich in potassium to the client. 4. Record the normal value on the intershift report sheet. 5. Gather data from the client related to signs of toxicity.

correct answer 2. Report the finding to the registered nurse. 5. Gather data from the client related to signs of toxicity. Rationale: The normal therapeutic range for digoxin is 0.5 to 2 ng/mL. A value of 2.7 ng/mL exceeds the therapeutic range and could be toxic to the client. The nurse should gather data about signs of digoxin toxicity (nausea/vomiting, seeing yellow rings) and then notify the registered nurse who will then contact the primary health care provider. The dose of digoxin should not be administered or recorded as normal. Foods high in potassium should not be administered without knowing the serum potassium level.

The nurse is assisting in caring for a client in the telemetry unit and is monitoring the client for cardiac changes indicative of hypokalemia. Which occurrence noted on the cardiac monitor indicates the presence of hypokalemia? 1. Tall, peaked T waves 2. ST-segment depression 3. Prolonged P-R interval 4. Widening of the QRS complex

correct answer 2. ST-segment depression Rationale: In the client with hypokalemia, the nurse would note ST-segment depression on a cardiac monitor. The client may also exhibit a flat T wave. Options 1, 3, and 4 are cardiac findings noted in the client with hyperkalemia.

The nurse is monitoring a client following cardioversion. Which observations would be of highest priority to the nurse? 1. Blood pressure 2. Status of airway 3. Oxygen flow rate 4. Level of consciousness

correct answer 2. Status of airway Rationale: Nursing responsibilities after cardioversion include maintenance of a patent airway, oxygen administration, assessment of vital signs and level of consciousness, and dysrhythmia detection. Airway is the priority.

The nurse assists in developing a plan of care for a client admitted to the hospital with an acute myocardial infarction (MI). Which is the priority problem during the acute phase? 1. The client's fear 2. The client's pain 3. The client's and family's concern 4. The client's home care support systems

correct answer 2. The client's pain Rationale: Pain is the prevailing symptom of acute MI. Relief of pain is a priority. Pain stimulates the autonomic nervous system, increasing myocardial oxygen demand. Although options 1, 3, and 4 are also appropriate problems, the presence of pain is the priority.

The nurse has completed nutritional counseling with an overweight client about weight reduction to modify the risk for coronary artery disease (CAD). The nurse would determine the teaching is successful if the client states that which weight loss goal is safe? 1. One half pound per day 2. Two pounds per week 3. Four pounds per week 4. Six pounds per week

correct answer 2. Two pounds per week Rationale: Most people, including the mildly and moderately obese, can lose only about 2 pounds per week of weight from fat loss. Weight loss beyond that level is probably due to protein and water loss alone.

A client with right-sided heart failure has been admitted to the hospital. The nurse is reviewing the medical record and notes which signs and symptoms? Select all that apply. 1. Orthopnea 2. Weight gain 3. Sleep apnea 4. Pitting edema 5. Pink frothy sputum

correct answer 2. Weight gain 4. Pitting edema Rationale: The signs and symptoms of right-sided heart failure include fatigue; edema in the sacrum, legs, feet, and ankles; hepatomegaly; abdominal distention as a result of ascites; weight gain; and dyspnea. Left-sided signs and symptoms include fatigue, dyspnea, wheezing, orthopnea, sleep apnea, pulmonary edema (pink, frothy sputum), pallor, and clammy skin.

A client taking an angiotensin-converting enzyme (ACE) inhibitor reviewed the medication information sheet and notes that the medication is used to treat hypertension. He states, "I have heart failure. Why am I taking this medicine?" The nurse responds by making which statement? 1. "There must be some mistake; I will check the medication prescriptions." 2."The medication causes relaxation in your arteries and veins and decreases the heart's work." 3. "The medication makes your heart beat faster, and this improves blood flow to your tissues." 4. "An additional medication will be added to the ACE inhibitor to strengthen your heart muscle."

correct answer 2."The medication causes relaxation in your arteries and veins and decreases the heart's work." Rationale: ACE inhibitors produce multiple benefits in heart failure. By lowering arteriolar tone, these medications improve regional blood flow, and by reducing cardiac afterload, they increase cardiac output. By causing venous dilation, they reduce pulmonary congestion and peripheral edema. By dilating blood vessels in the kidney, they increase renal blood flow and thereby promote excretion of sodium and water. This loss of fluid has two beneficial effects: (1) it helps reduce edema and (2) by lowering blood volume, it decreases venous return to the heart thereby reducing right-heart size. Also by suppressing aldosterone and by reducing local production of angiotensin II in the heart, ACE inhibitors may prevent or reverse pathological changes in cardiac structure. Therefore, options 1, 3, and 4 are incorrect.

To use an external cardiac defibrillator on a client, which action would be performed to check the cardiac rhythm? 1. Holding the defibrillator paddles firmly against the chest 2.Applying the adhesive patch electrodes to the skin and moving away from the client 3. Applying standard electrocardiographic monitoring leads to the client and observing the rhythm 4. Connecting standard electrocardiographic electrodes to a transtelephonic monitoring device

correct answer 2.Applying the adhesive patch electrodes to the skin and moving away from the client Rationale: The nurse or rescuer puts two large adhesive patch electrodes on the client's chest in the usual defibrillator position. The nurse stops cardiopulmonary resuscitation and orders anyone near the client to move away and not touch the client. The defibrillator then analyzes the rhythm, which may take up to 30 seconds. The machine then indicates if it is necessary to defibrillate. Although automatic external defibrillation can be done transtelephonically, it is done through the use of patch electrodes (not standard electrocardiographic electrodes) that interact via telephone lines to a base station that controls any actual defibrillation. It is not necessary to hold defibrillator paddles against the client's chest with this device.

A client is being treated for moderate hypertension and has been taking diltiazem for several months. The client is seen by the primary health care provider, and Prinzmetal's angina is diagnosed. The nurse is instructing the client about diltiazem. Which client statement indicates a need for further teaching? 1. "I have to be careful getting out of my recliner." 2"I need to avoid hazardous activities until I don't get dizzy." 3"I have to limit my coffee, but I can drink all the fruit juice I want." 4"I will take and record my blood pressure and pulse every morning."

correct answer 3"I have to limit my coffee, but I can drink all the fruit juice I want." Rationale: Diltiazem is a calcium-channel blocker that inhibits calcium influx through the slow channels of the membrane of smooth muscle cells. Calcium-channel blockers decrease myocardial oxygen demands and block calcium channels, thereby decreasing the force of contraction of the ventricular tissue. Client teaching about diltiazem includes taking pulse and blood pressure and keeping a record, changing position slowly, avoiding hazardous activities until stabilized, limiting caffeine consumption, and avoiding grapefruit juice.

The nurse has reinforced dietary instructions to a client with coronary artery disease. Which statement by the client indicates an understanding of the dietary instructions? 1. "I need to substitute eggs and milk for meat." 2. "I will eliminate all cholesterol and fat from my diet." 3. "I should routinely use polyunsaturated oils in my diet." 4. "I need to seriously consider becoming a strict vegetarian."

correct answer 3. "I should routinely use polyunsaturated oils in my diet." Rationale: The client with coronary artery disease should avoid foods high in saturated fat and cholesterol such as eggs, whole milk, and red meat. These foods contribute to increases in low-density lipoproteins. The use of polyunsaturated oils is recommended to control hyperlipidemia. It is not necessary to eliminate all cholesterol and fat from the diet. It is not necessary to become a strict vegetarian.

While the nurse is involved in preparing a client for a cardiac catheterization, the client says, "I don't want to talk with you. You're only the nurse. I want my doctor." Which response by the nurse would be therapeutic? 1. "Your doctor expects me to prepare you for this procedure." 2. "That's fine, if that's what you want. I'll call your primary health care provider." 3. "So you're saying that you want to talk to your primary health care provider?" 4. "I'm concerned with the way you've dismissed me. I know what I am doing."

correct answer 3. "So, you're saying that you want to talk to your primary health care provider?" Rationale: In option 3, the nurse uses the therapeutic communication technique of reflection to redirect the client's feelings back for validation. Option 1 is nontherapeutic and addresses the legal issue of performing a procedure when in fact the client is refusing. Although option 2 may seem appropriate, it does not reflect the client's feelings and doesn't provide an opportunity for the client to express feelings. Option 4 is clearly nontherapeutic because it focuses on the nurse's feelings rather than the client's feelings.

The nurse is reinforcing discharge teaching to a client who was given a prescription for nifedipine for blood pressure management. Which instructions would the nurse reinforce? Select all that apply. 1. "Increase water intake." 2."Increase calcium intake." 3. "Take pulse rate each day." 4. "Weigh at the same time each day." 5. "Palpitations may occur early in therapy." 6. "Be careful when rising from sitting to standing."

correct answer 3. "Take pulse rate each day." 4. "Weigh at the same time each day." 5. "Palpitations may occur early in therapy." 6. "Be careful when rising from sitting to standing." Rationale: Nifedipine is a calcium-channel blocker. Its therapeutic outcome is to decrease blood pressure. Its method of action is blockade of the calcium channels in vascular smooth muscle, promoting vasodilation. Side effects that can occur early in therapy include reflex tachycardia (palpitations) and first-dose hypotension, leading to orthostatic hypotension. Weight should be checked regularly to monitor for early signs of heart failure. Also the client is taught to take his or her own pulse. Nifedipine does not affect serum calcium levels. Increased water intake is not indicated in the client with cardiovascular disease.

An antihypertensive medication has been prescribed for a client with hypertension. The client tells the nurse that she would like to take an herbal substance to help lower her blood pressure. Which statement by the nurse is most important to provide to the client? 1. "Herbal substances are not safe and should never be used." 2. "I will teach you how to take your blood pressure so that it can be monitored closely." 3. "You will need to talk to your primary health care provider (PHCP) before using an herbal substance." 4. "If you take an herbal substance, you will need to have your blood pressure checked frequently."

correct answer 3. "You will need to talk to your primary health care provider (PHCP) before using an herbal substance." Rationale: Although herbal substances may have some beneficial effects, not all herbs are safe to use. Clients who are being treated with conventional medication therapy should be advised to avoid herbal substances with similar pharmacological effects, because the combination may lead to an excessive reaction or unknown interaction effects. Therefore, the nurse would advise the client to discuss the use of the herbal substance with the PHCP.

An automatic external defibrillator (AED) is available to treat a client who goes into cardiac arrest. The nurse uses this equipment to determine cardiac rhythm by doing which? 1. Holding the defibrillator paddles firmly against the chest 2. Applying standard ECG monitoring leads to the client and observing the rhythm 3. Applying the adhesive patch electrodes to the skin and moving away from the client 4. Connecting standard electrocardiographic (ECG) electrodes to a transtelephonic monitoring device

correct answer 3. Applying the adhesive patch electrodes to the skin and moving away from the client Rationale: The nurse or rescuer puts two large adhesive patch electrodes on the client's chest in the usual defibrillator positions. The nurse orders anyone near the client to move away and not touch the client (to eliminate movement artifact). The defibrillator analyzes the rhythm, which may take up to 30 seconds. The machine then indicates if it is necessary to defibrillate.

The nurse is assisting in the care of a client with myocardial infarction who should reduce intake of saturated fat and cholesterol. The nurse would help the client comply with diet therapy by selecting which food items from the dietary menu? 1. Cheeseburger, pan-fried potatoes, whole kernel corn, sherbet 2. Pork chop, baked potato, cauliflower in cheese sauce, ice cream 3. Baked haddock, steamed broccoli, herbed rice, sliced strawberries 4. Spaghetti and sweet sausage in tomato sauce, vanilla pudding (with 4% milk)

correct answer 3. Baked haddock, steamed broccoli, herbed rice, sliced strawberries Rationale: A client trying to lower fat and cholesterol in the diet should decrease the use of fatty cuts of meats such as beef, lamb or pork, organ meats, sausage, hot dogs, bacon, and sardines; avoid vegetables prepared in butter, cream, or other sauces; use low-fat milk products instead of whole milk products and cream; and decrease the amount of commercially prepared baked goods. Option 3 is the only option that identifies low-fat and low-cholesterol foods.

A client with a history of hypertension has been prescribed triamterene. The nurse determines that the client understands the effect of this medication on the diet if the client states to avoid which fruit? 1. Pears 2. Apples 3. Bananas 4. Cranberries

correct answer 3. Bananas Rationale: Triamterene is a potassium-retaining diuretic, and the client should avoid foods high in potassium. Fruits that are naturally higher in potassium include avocados, bananas, fresh oranges and mangoes, nectarines, papayas, and dried prunes and other dried fruits.

A client is diagnosed with thrombophlebitis. The nurse would tell the client that which prescription is indicated? 1. Bed rest with bathroom privileges only 2. Bed rest keeping the affected extremity flat 3. Bed rest with elevation of the affected extremity 4. Bed rest with the affected extremity in a dependent position

correct answer 3. Bed rest with elevation of the affected extremity Rationale: Elevation of the affected leg facilitates blood flow by the force of gravity and decreases venous pressure, which in turn relieves edema and pain. The foot of the bed is elevated, and bed rest is indicated to prevent emboli and pressure fluctuations in the venous system that occur with walking. The positions in the remaining options are incorrect.

A client with angina pectoris is experiencing chest pain that radiates down the left arm. The nurse administers a sublingual nitroglycerin tablet to the client. The client's pain is unrelieved, and the nurse determines that the client needs another nitroglycerin tablet. Which vital sign is most important for the nurse to check before administering a second dose of the medication? 1. Temperature 2. Respirations 3. Blood pressure 4.Radial pulse rate

correct answer 3. Blood pressure Rationale: Nitroglycerin acts directly on the smooth muscle of the blood vessels, causing relaxation and dilation. As a result, hypotension can occur. The nurse should check the client's blood pressure before administering the second nitroglycerin tablet. Although the respirations, apical pulse, and temperature may be checked, these vital signs are not the most important assessments related to administration of this medication.

The nurse is caring for a client on a cardiac monitor who is alone in a room at the end of the hall. The client has a short burst of ventricular tachycardia (VT), followed by ventricular fibrillation (VF). The client suddenly loses consciousness. Which intervention would the nurse do first? 1. Go to the nurse's station quickly and call a code. 2. Run to get a defibrillator from an adjacent nursing unit. 3. Call for help and initiate cardiopulmonary resuscitation (CPR). 4. Start oxygen by cannula at 10 L/minute and lower the head of the bed.

correct answer 3. Call for help and initiate cardiopulmonary resuscitation (CPR). Rationale: When ventricular fibrillation occurs, the nurse remains with the client and initiates CPR until a defibrillator is available and attached to the client. Options 1, 2, and 4 are incorrect.

A client returning to the medical nursing unit following cardiac catheterization has a stat prescription to receive a dose of procainamide. The licensed practical nurse assisting in caring for the client obtains which piece of equipment to adequately determine the client's response to this medication? 1. Glucometer 2. Pulse oximeter 3. Cardiac monitor 4. Noninvasive blood pressure cuff

correct answer 3. Cardiac monitor Rationale: Procainamide is an antidysrhythmic medication often used to treat ventricular dysrhythmias that do not adequately respond to lidocaine. The effectiveness of this medication is best determined by evaluating the client's cardiac rhythm. Therefore, a cardiac monitor is of greatest value, although the blood pressure cuff and pulse oximeter would provide general information about the client's cardiovascular status. A glucometer is not needed for this client with the information presented.

A client returning to the medical nursing unit following cardiac catheterization has a stat prescription to receive a dose of procainamide. The licensed practical nurse assisting in caring for the client obtains which piece of equipment to adequately determine the client's response to this medication? 1. Glucometer 2. Pulse oximeter 3. Cardiac monitor 4. Noninvasive blood pressure cuff

correct answer 3. Cardiac monitor Rationale: Procainamide is an antidysrhythmic medication often used to treat ventricular dysrhythmias that do not adequately respond to lidocaine. The effectiveness of this medication is best determined by evaluating the client's cardiac rhythm. Therefore, a cardiac monitor is of greatest value, although the blood pressure cuff and pulse oximeter would provide general information about the client's cardiovascular status. A glucometer is not needed for this client with the information presented.

A hospitalized client with coronary artery disease complains of substernal chest pain. After checking the client's heart rate and blood pressure, the nurse administers nitroglycerin, 0.4 mg, sublingually. After 5 minutes, the client states, "My chest still hurts." Which appropriate actions would the nurse take? Select all that apply. 1. Call a code blue. 2. Contact the client's family. 3. Check the client's pain level. 4. Check the client's blood pressure. 5. Administer a second nitroglycerin, 0.4 mg, sublingually.

correct answer 3. Check the client's pain level. 4. Check the client's blood pressure. 5. Administer a second nitroglycerin, 0.4 mg, sublingually. Rationale: The usual guideline for administering nitroglycerin tablets for a hospitalized client with chest pain is to administer one tablet every 5 minutes as needed (PRN) for chest pain for a total dose of 3 tablets. The registered nurse is notified immediately if a client complains of chest pain. In this situation, because the client is still complaining of chest pain, the nurse would administer a second nitroglycerin tablet. The nurse would check the client's pain level and the client's blood pressure before administering each nitroglycerin dose. There are no data in the question that indicate the need to call a code blue. In addition, it is not necessary to contact the client's family unless the client has requested this.

The nurse determines that which client is most likely to be a candidate for cardioversion? 1. Client with junctional rhythm 2. Client with ventricular fibrillation 3. Client with unstable rapid atrial fibrillation 4. Client with pulseless ventricular tachycardi

correct answer 3. Client with unstable rapid atrial fibrillation Rationale: Cardioversion is a synchronized shock, delivered during ventricular depolarization. The machine must be able to seek out R waves and mark them so that the device delivers the shock at the appropriate time. Clients in atrial fibrillation are candidates for this treatment, and the goal is to try to restore normal sinus rhythm through cardioversion. Although the client with ventricular tachycardia can be cardioverted because of the presence of QRS complexes, this is done only when the client has a pulse. Pulseless ventricular tachycardia and ventricular fibrillation clients always are defibrillated. Junctional rhythm is neither cardioverted nor defibrillated.

A client has experienced an episode of pulmonary edema. The nurse determines that the client's respiratory status is improving if which breath sounds are noted? 1. Rhonchi 2. Wheezes 3. Crackles in the lung bases 4. Crackles throughout the lung fields

correct answer 3. Crackles in the lung bases Rationale: Pulmonary edema is characterized by extreme breathlessness, dyspnea, air hunger, and the production of frothy pink-tinged sputum. Auscultation of the lungs reveals crackles throughout the lung fields. As the client's condition improves, the amount of fluid in the alveoli decreases and may be detected by crackles in the bases. (Clear lung sounds would indicate full resolution of the episode.) Wheezes and rhonchi are not associated with pulmonary edema.

An older client has been prescribed digoxin. The nurse determines that which age-related change would place the client at risk for digoxin toxicity? 1. Decreased salivation and gastrointestinal motility 2. Decreased muscle strength and loss of bone density 3. Decreased lean body mass and glomerular filtration rate 4. Decreased cardiac output and decreased efficiency of blood return to the heart

correct answer 3. Decreased lean body mass and glomerular filtration rate Rationale: The older client is at risk for medication toxicity because of decreased lean body mass and an age-associated decreased glomerular filtration rate. Although the other changes identify age-related changes that occur in the older client, they are not specifically associated with this risk.

The nurse is collecting data on a client with a diagnosis of right-sided heart failure. The nurse would expect to note which specific characteristic of this condition? 1. Dyspnea 2. Hacking cough 3. Dependent edema 4. Crackles on lung auscultation

correct answer 3. Dependent edema Rationale: Right-sided heart failure is characterized by signs of systemic congestion that occur as a result of right ventricular failure, fluid retention, and pressure buildup in the venous system. Edema develops in the lower legs and ascends to the thighs and abdominal wall. Other characteristics include jugular (neck vein) congestion, enlarged liver and spleen, anorexia and nausea, distended abdomen, swollen hands and fingers, polyuria at night, and weight gain. Left-sided heart failure produces pulmonary signs. These include dyspnea, crackles on lung auscultation, and a hacking cough.

A female client complains of an "odd, left-sided, twinge-like pain" along the anterior axillary line and states she has had this feeling for the past 3 days. Which is the initial action? 1. Administer naproxen. 2. Listen to the client's heart and lungs. 3. Determine if the pain is cardiac in origin. 4. Ask the client about previous cardiac disease.

correct answer 3. Determine if the pain is cardiac in origin. Rationale: The best initial action is to rule out chest pain of cardiac origin to eliminate a cardiovascular etiology related to the client's complaint. If the pain is left untreated and the pain is caused by myocardial ischemia or infarction (MI), the client could suffer a devastating cardiac injury. Furthermore, the nurse does this because a female presenting with an MI is more likely to display atypical clinical indicators including fatigue and dyspnea. After instituting measures to rule out a cardiac problem, the nurse completes the client assessment by auscultating the heart and lungs and by reviewing the medical record. After a cardiac problem is ruled out, the nurse can administer an analgesic if prescribed.

A client complaining of not feeling well is seen in a clinic. The client is taking several medications for the control of heart disease and hypertension. These medications include a beta blocker, digoxin, and a diuretic. A tentative diagnosis of digoxin toxicity is made. Which assessment data supports this diagnosis? 1. Dyspnea, edema, and palpitations 2. Chest pain, hypotension, and paresthesia 3. Double vision, loss of appetite, and nausea 4. Constipation, dry mouth, and sleep disorder

correct answer 3. Double vision, loss of appetite, and nausea Rationale: Double vision, loss of appetite, and nausea are signs of digoxin toxicity. Additional signs of digoxin toxicity include bradycardia, difficulty reading, visual alterations such as green and yellow vision or seeing spots or halos, confusion, vomiting, diarrhea, decreased libido, and impotence.

The nurse monitors the laboratory data on a client at risk for coronary artery disease. A fasting blood glucose reading of 200 mg/dL is recorded on the chart. The nurse analyzes this result as indicative of which finding? 1. Decreased, indicating a decreased risk of coronary artery disease 2. Elevated, but would not present a risk for coronary artery disease 3. Elevated, signaling the presence of diabetes mellitus, a risk factor of coronary artery disease 4. Normal, indicating adequate blood glucose control with no risk for coronary artery disease

correct answer 3. Elevated, signaling the presence of diabetes mellitus, a risk factor of coronary artery disease Rationale: A fasting blood glucose of 200 mg/dL signals the presence of diabetes mellitus. Diabetes mellitus predisposes a client to coronary artery disease. Options 1, 2, and 4 are inaccurate interpretations.

The nurse observes the following rhythm on the cardiac monitor. Which action would the nurse take first? Refer to the figure. 1. Notify the primary health care provider. 2. Prepare for transcutaneous pacing. 3. Evaluate the client for hypotension and assess mental status. 4. Carry out protocol prescriptions for the administration of atropine.

correct answer 3. Evaluate the client for hypotension and assess mental status. Rationale: After determining that the client is in sinus bradycardia with a heart rate of 40 beats per minute, the nurse should evaluate the client first for signs and symptoms of decreased cardiac output. Signs and symptoms of decreased cardiac output include hypotension, altered mental status, weak peripheral pulses, and decreased urinary output. If the bradycardia is new or the client has evidence of decreased cardiac output, the primary health care provider would then be notified. Transcutaneous pacing or atropine administration may be instituted if evidence of decreased cardiac output is present.

A client who experienced a myocardial infarction (MI) tells the nurse that he is fearful about not being able to return to a normal life. Which action by the nurse is therapeutic at this time? 1. Tell the client that his fears are not rational. 2. Tell the client that his life has not changed. 3. Explore specific concerns with the client. 4. Tell the client to talk it out with the significant other.

correct answer 3. Explore specific concerns with the client. Rationale: The therapeutic action by the nurse is one that gathers more data. This then allows the nurse to formulate the appropriate response. Each of the incorrect options is nontherapeutic because they place the client's feelings on hold and do not address them.

A client is admitted to the hospital with possible rheumatic heart disease. The nurse collects data from the client and checks the client for which signs/symptoms? 1. Skin scratches 2. Vaginal itching 3. Fever and sore throat 4. Burning on urination

correct answer 3. Fever and sore throat Rationale: Rheumatic heart disease can occur as a result of infection with group A beta-hemolytic streptococcal infections. It is frequently triggered by streptococcal pharyngitis, which is assessed by noting for the presence of sore throat and fever. The other options are unrelated to this problem and indicate possible yeast infection, skin lesions, and urinary tract infection, respectively.

While collecting data related to the cardiac system on a client, the nurse hears a murmur. Which best describes the sound of a heart murmur? 1. Lub-dub sounds 2. Scratchy, leathery heart noise 3. Gentle, blowing or swooshing noise 4. Abrupt, high-pitched snapping noise

correct answer 3. Gentle, blowing or swooshing noise Rationale: A heart murmur is an abnormal heart sound and is described as a gentle, blowing, swooshing sound. It occurs from increased or abnormal blood flow through the valves of the heart. Lub-dub sounds are normal and represent the S1 (first heart sound) and S2 (second heart sound), respectively. A pericardial friction rub is described as a scratchy, leathery heart sound that occurs with pericarditis. A click is described as an abrupt, high-pitched snapping sound.

The nurse administering medications to a client notes a prescription to give a subcutaneous dose of heparin sodium. The nurse would perform which action to give this medication safely? 1. Massage the site after injection. 2. Aspirate with the plunger before injecting. 3. Give the injection using a 25- to 27-gauge, ½-inch needle. 4. Withdraw medication using a 1½-inch needle and then change to a 1-inch needle.

correct answer 3. Give the injection using a 25- to 27-gauge, ½-inch needle. Rationale: Subcutaneous heparin sodium is given using a 25- to 27-gauge, ½-inch needle to prevent tissue trauma and inadvertent injection into muscle. (A 1-inch needle could inject the heparin sodium into the muscle.) The nurse does not aspirate or massage to prevent tissue trauma and bleeding.

The nurse is assigned to care for a group of clients on the clinical nursing unit. Which client is least likely to develop third spacing of fluids? 1. Major burn 2. Renal failure 3. Hypertension 4. Laënnec's cirrhosis

correct answer 3. Hypertension Rationale: Fluid that shifts into the interstitial spaces and remains there is referred to as "third-space fluid." This fluid is physiologically useless because it does not circulate to provide nutrients for the cells. Common sites for third spacing include body tissues, the pleural and peritoneal cavities and the pericardial sac. In clients with severe burns, fluid shifts out to the tissues in the area of the burn as sometimes evidenced by blistering. In clients with renal failure, there is a loss of protein due to failure of the kidney to retain protein, and fluid shifts from the blood out into the tissues causing the client to have edema in extremities and face. In Laënnec's cirrhosis the liver becomes fibrotic because of insufficient protein intake, alcoholism, and other conditions. The liver normally produces protein as albumin. With the loss of sufficient levels of albumin in clients with cirrhosis, the fluid shifts out into the abdomen (ascites) or tissues. Hypertension, elevated blood pressure, by itself is not a mechanism leading to fluid shifts. Risk factors for third spacing of fluids include the older client, and those with liver or kidney disease, major trauma, burns, sepsis, major surgery, malignancy, and gastrointestinal malabsorption and malnutrition.

The nurse is providing information to the family of a client recovering from a cardiac arrest. Which information made by the family indicates a need for follow-up? 1. Temperature will be watched to help with brain perfusion. 2. Medications will be used to support blood pressure and heart rate. 3. Intravenous fluids will be stopped to decrease excess fluid buildup. 4. Airway management will be maintained to help with respiratory support.

correct answer 3. Intravenous fluids will be stopped to decrease excess fluid buildup. Rationale: The recovery phase following a cardiac arrest is a critical period in which tissue and organ perfusion must be watched carefully. Treatment and management focus on maintaining a patent airway and managing respiratory support, supporting recovery of the heart by managing the heart rate and blood pressure, protecting the perfusion of the brain by keeping the body's temperature normothermic or lower, and resuscitating the circulating volume with intravenous fluids and blood product.

The nurse has given simple instructions on preventing some of the complications of bed rest to a client who experienced a myocardial infarction. The nurse would intervene if the client were performing which of these contraindicated activities? 1. Deep breathing and coughing 2. Repositioning self from side to side 3. Isometric exercises of the arms and legs 4. Ankle circles, plantar, and dorsiflexion exercises

correct answer 3. Isometric exercises of the arms and legs Rationale: The client with myocardial infarction should avoid activities that tense the muscles, such as isometric exercises. These increase intra-abdominal and intrathoracic pressures and can decrease the cardiac output. They also can trigger vagal stimulation causing bradycardia. Deep breathing and coughing, repositioning self from side to side, and ankle circles, plantar, and dorsiflexion exercises are acceptable.

A client is diagnosed with an acute myocardial infarction and is receiving tissue plasminogen activator, alteplase. Which action is a priority nursing intervention? 1. Monitor for kidney failure. 2. Monitor psychosocial status. 3. Monitor for signs of bleeding. 4. Have heparin sodium available.

correct answer 3. Monitor for signs of bleeding. Rationale: Tissue plasminogen activator is thrombolytic. Hemorrhage is a complication of any type of thrombolytic medication. The client is monitored for bleeding. Monitoring for renal failure and monitoring the client's psychosocial status are important but are not the most critical interventions. Heparin is given after thrombolytic therapy, but the question is not asking about follow-up medications.

A client is being treated for acute heart failure with intravenously administered bumetanide. The vital signs are as follows: blood pressure, 100/60 mm Hg; pulse, 96 beats per minute; and respirations, 24 breaths per minute. After the initial dose, which is the priority assessment? 1. Monitoring weight loss 2. Monitoring temperature 3. Monitoring blood pressure 4. Monitoring potassium level

correct answer 3. Monitoring blood pressure Rationale: Bumetanide is a loop diuretic. Hypotension is a common side effect associated with the use of this medication. The other options also require assessment but are not the priority.

The nurse is assisting in caring for a client who is receiving an intravenous infusion of 1000 mL of normal saline with 40 mEq of potassium chloride. The nurse is monitoring the client for signs of hyperkalemia. Which sign/symptom would be noted in the client if hyperkalemia is present? 1. Muscle pain 2. Mental confusion 3. Muscle weakness 4. Depressed deep tendon reflexes

correct answer 3. Muscle weakness Rationale: Because potassium plays a major role in neuromuscular activity, elevation in serum potassium initially causes muscle weakness, not muscle pain. Mental status changes and confusion are most likely noted in the client experiencing hypocalcemia hyponatremia. Depressed deep tendon reflexes are noted in the client with hypermagnesemia.

The nurse is caring for a client diagnosed with Buerger's disease. Which finding would the nurse determine is a potential complication associated with this disease? 1. Pain with diaphoresis 2. Discomfort in one digit 3. Numbness and tingling in the legs 4. Cramping in the foot while resting

correct answer 3. Numbness and tingling in the legs Rationale: Buerger's disease (thromboangiitis obliterans), which affects men between 20 and 40 years of age, has an unknown etiology. It is a recurring inflammation of the small and medium-sized arteries and veins of the upper and lower extremities that results in thrombus formation and occlusion of blood vessels. Pain with diaphoresis, discomfort in one digit, and cramping in the foot are not complications of this disorder. The finding that can be interpreted as a complication of the disorder is numbness and tingling in the legs.

The nurse is setting up the bedside unit for a client being admitted to the nursing unit from the emergency department with a diagnosis of coronary artery disease (CAD). The nurse would place highest priority on making sure that which is available at the bedside? 1. Bedside commode 2. Rolling shower chair 3. Oxygen tubing and flowmeter 4. Twelve-lead electrocardiogram (ECG) machine

correct answer 3. Oxygen tubing and flowmeter Rationale: CAD causes obstruction to blood flow through one or more major coronary arteries, cutting off oxygen and nutrients to the cardiac cells, and resulting in chest pain. Providing oxygen to the client is important to help decrease pain and prevent its recurrence. A bedside commode and ECG machine may be helpful but are not the priority. A rolling shower chair has no value for this client because the client should be able to walk and shower if pain free and an activity prescription allows it.

A client is receiving standard oral anticoagulant therapy with warfarin. The result of a newly drawn international normalized ratio is 3.8 seconds. The client needs to have an invasive procedure done on the next day. Which medication will likely be ordered to reverse the anticoagulant effect? 1. Naloxone 2. Protamine 3. Phytonadione 4. Calcium chloride

correct answer 3. Phytonadione Rationale: A client who is prescribed warfarin will need to have the medication reversed if an invasive procedure is planned. Vitamin K1 or phytonadione is administered to reverse warfarin. Naloxone reverses narcotics. Protamine reverses heparin. Calcium chloride will not reverse warfarin.

The nurse is monitoring a client who is attached to a cardiac monitor and notes the presence of U waves. The nurse checks the client and then reviews the results of the client's recent electrolyte results. The nurse expects to note which electrolyte value? 1. Sodium 135 mEq/L 2. Sodium 140 mEq/L 3. Potassium 3.0 mEq/L 4. Potassium 5.0 mEq/L

correct answer 3. Potassium 3.0 mEq/L Rationale: The U wave is a very small wave that may be present following the T wave on a heart monitor strip. It is thought to represent repolarization of the Purkinje fibers. It is present in some clients who have hypokalemia. A serum potassium level below 3.5 mEq/L is indicative of hypokalemia. In hypokalemia, the electrocardiogram (ECG) changes that occur include inverted T waves, ST segment depression, heart block, and prominent U waves. The normal sodium level is 135 to 145 mEq/L. The normal potassium level is 3.5 to 5.0 mEq/L.

A hypertensive client who has been taking metoprolol has been prescribed to decrease the dose of the medication. The client asks the nurse why this must be done over a period of 1 to 2 weeks. In formulating a response, the nurse incorporates the understanding that abrupt withdrawal could affect the client in which way? 1. Result in hypoglycemia 2. Give the client insomnia 3. Precipitate rebound hypertension 4. Cause enhanced side effects of other prescribed medications

correct answer 3. Precipitate rebound hypertension Rationale: Beta-adrenergic blocking agents should be tapered slowly. This will avoid abrupt withdrawal syndrome characterized by headache, malaise, palpitations, tremors, sweating, rebound hypertension, dysrhythmias, and possibly myocardial infarction (in clients with cardiac disorders including angina pectoris). Options 1, 2, and 4 are incorrect.

The health care team is performing high-quality cardiopulmonary resuscitation (CPR) on a client in cardiac arrest. Once the client is attached to a monitor, it is determined that a shock is not advised. CPR is continued, and the nurse determines what additional action would be taken next by the health care team? 1. Consider reversible causes. 2. Prepare the client for intubation 3. Prepare to administer epinephrine. 4. Monitor until the rhythm becomes shockable.

correct answer 3. Prepare to administer epinephrine. Rationale: For non-shockable rhythms there are algorithms that are followed to get a pulse and rhythm back. Continuing CPR is priority, followed by administering epinephrine every 3-5 minutes. An airway should be established, and potential reversible causes should be investigated. Should a rhythm and pulse come back, the monitor will continue to identify if it is shockable or not.

A client has been diagnosed with Prinzmetal's angina. The nurse reviews the medical record and notes which accompanying characteristics? Select all that apply. 1. Relieved by rest 2. Occurs after exercise 3. Prolonged severe pain 4. Nitroglycerine relieves the pain 5. Happens at the same time each day

correct answer 3. Prolonged severe pain 5. Happens at the same time each day Rationale: Variant angina, or Prinzmetal's angina, is prolonged and severe and occurs at the same time each day, most often in the morning. Stable angina is induced by exercise and relieved by rest or nitroglycerin tablets.

Warfarin sodium is prescribed for a client. The nurse expects that the primary health care provider will prescribe which laboratory tests to monitor for a therapeutic effect of the medication? Select all that apply. 1. D-dimer 2. Platelet count 3. Prothrombin time (PT) 4. International normalized ratio (INR) 5. Activated partial thromboplastin time (aPTT)

correct answer 3. Prothrombin time (PT) 4. International normalized ratio (INR) Rationale: The INR will assess for the therapeutic effect of warfarin sodium, and the aPTT will assess the therapeutic effect of heparin sodium. The platelet count will assess the client's potential for bleeding. Warfarin sodium doses are determined based on the results of the INR. The PT also is used to determine warfarin dosage. The D-dimer is used to detect pulmonary embolism and disseminated intravascular coagulation (DIC). The APTT is used to determine heparin dosage.

The nurse is performing an abdominal assessment on a client. The nurse interprets that which finding is abnormal and would be immediately reported? 1. Absence of a bruit 2. Concave, midline umbilicus 3. Pulsation between the umbilicus and pubis 4. Bowel sound frequency of 15 sounds per minute

correct answer 3. Pulsation between the umbilicus and pubis Rationale: The umbilicus should be in the midline with a concave appearance. The presence of pulsation between the umbilicus and the pubis could indicate an abdominal aortic aneurysm and should be reported. Bruits are not normally present. Bowel sounds vary according to the timing of the last meal and usually range in frequency from 5 to 35 per minute.

A pregnant client with severe uterine bleeding is admitted to the labor and birthing department. Which data would best alert the nurse to early signs of hypovolemic shock? 1. Cold and clammy skin 2. Decreased blood pressure 3. Restlessness and agitation 4. Diminished peripheral pulses

correct answer 3. Restlessness and agitation Rationale: Early signs of hypovolemic shock include restlessness, anxiety, and agitation. Later signs of hypovolemic shock include cold and clammy skin, a falling blood pressure, diminished peripheral pulses, pallor, and urine output less than 30 mL/hr. "Restlessness and agitation" is the correct answer option.

A client is admitted to the hospital with a venous stasis leg ulcer. The nurse is reviewing the physical examination in the medical record. Which symptom is least likely to be noted by the primary health care provider? 1. Itchy scaly skin 2. Lower leg edema 3. Sharp pain in the leg 4. Brownish skin discoloration

correct answer 3. Sharp pain in the leg Rationale: Characteristics of a venous stasis ulcer include lower leg edema, itchy scaly skin, brownish discoloration, and pain described as heaviness or dull ache in the calf or thigh.

The nurse is admitting a client with acute pericarditis who reports chest pain. When planning the client's care, which positions would the nurse encourage the client to assume to alleviate the chest pain? Select all that apply. 1. Lying supine 2. Right side-lying 3. Sitting up and leaning forward 4. Semi-Fowler's with knees bent 5. Head of bed elevated to 45 degrees

correct answer 3. Sitting up and leaning forward 5. Head of bed elevated to 45 degrees Rationale: Acute pericarditis refers to inflammation of the pericardial sac. A common symptom is chest pain. Chest pain is often relieved when the client sits up and leans forward or has the head of bed elevated to 45 degrees. Lying supine makes the pain worse. Right side-lying and semi-Fowler's with knees bent do not relieve the chest pain associated with acute pericarditis.

A physician tells the nurse that a potassium-retaining diuretic is being prescribed for the client with heart failure. The nurse reviews the health care physician's prescriptions expecting that which medication will be prescribed? 1. Furosemide 2. Ethacrynic acid 3. Spironolactone 4. Hydrochlorothiazide

correct answer 3. Spironolactone Rationale: Spironolactone is a potassium-retaining diuretic that promotes sodium excretion while conserving potassium. Options 1, 2, and 4 identify diuretics that do not conserve potassium.

The nurse is caring for a new postoperative client and is monitoring the client for signs of shock. The nurse monitors for which signs of this postoperative complication? 1. Cold skin, drowsiness, and hypertension 2. Fever, irritability, and rapid respirations 3. Tachycardia, cold skin, and hypotension 4. Slow pulse, warm skin, and restlessness

correct answer 3. Tachycardia, cold skin, and hypotension Rationale: Postoperative hypotension or shock can have numerous causes such as inadequate ventilation, side effects of anesthetic agents or preoperative medications, and fluid or blood loss. The symptoms of shock include hypotension; tachycardia; cold, moist, pale, or cyanotic skin; and increased restlessness and apprehension.

The nurse is caring for a client with severe cardiac disease. While the nurse is caring for the client, the client states, "If anything should happen to me, please make sure that the doctors do not try to push on my chest and revive me." Which is the appropriate nursing action? 1. Tell the client that the family must agree with the request. 2. Plan a client conference with the nursing staff to share the client's request. 3. Tell the client that it is necessary to notify the primary health care provider of the client's request. . 4Tell the client that this procedure cannot legally be refused by a client if the primary health care provider feels that it is necessary to save the client's life.

correct answer 3. Tell the client that it is necessary to notify the primary health care provider of the client's request. Rationale: External cardiac massage is one type of treatment that a client can refuse. The most appropriate nursing action is to notify the primary health care provider because a written "do not resuscitate" (DNR) order from the primary health care provider must be present on the client's record. The DNR order must be reviewed or renewed on a regular basis per agency policy. Options 1 and 4 are inaccurate. Option 2 may be appropriate, but only after the primary health care provider is contacted and notified of the client's request.

The nurse is collecting data from a client with varicose veins. Which finding would the nurse identify as an indication of a potential complication associated with this disorder? 1. Legs are unsightly in appearance and distress the client. 2. The client complains of aching and feelings of heaviness in the legs. 3. The client complains of leg edema, and skin breakdown has started. 4. The legs become distended when the tourniquet is released during the Trendelenburg's test.

correct answer 3. The client complains of leg edema, and skin breakdown has started. Rationale: Complications of varicose veins include leg edema, skin breakdown, ulceration of the legs, trauma leading to rupture of a varicosity, deep vein thrombosis, or chronic insufficiency. The client with varicose veins may be distressed about the unsightly appearance of the varicosities. Complaints of heaviness and aching in the legs are common. Option 4 describes the Trendelenburg's test findings, which are indicative of varicose veins. In the test, the health care provider has the client lie down and elevate the legs to empty the veins. A tourniquet is then applied to occlude the superficial veins, after which the client stands and the tourniquet is released. If the veins are incompetent, they will quickly become distended due to backflow.

The nurse is talking to the family of a hospitalized client recovering from a cardiac arrest. What information is best for the nurse to plan to share with the family? 1. The recovery process is long, and family support is crucial. 2. The client has a high incidence of recurring cardiac arrest. 3. The critical care unit will help manage coronary reperfusion. 4. The longer the client needs medications, the more the chance of recovery decreases.

correct answer 3. The critical care unit will help manage coronary reperfusion. Rationale: Therapeutic communication is a part of creating a client-centered care environment. The goal of communication is to not speak out of the nurse's scope of practice, to instill false hope, or to give wrong information. Options 1, 2, and 4 provide inaccurate information.

The nurse working in a long-term care facility is collecting data from a client experiencing chest pain. The nurse would interpret that the pain is likely a result of myocardial infarction (MI) if which observation is made by the nurse? 1. The client is not experiencing nausea or vomiting. 2. The pain is described as substernal and radiating to the left arm. 3. The pain has not been relieved by rest and nitroglycerin tablets. 4. The client says the pain began while trying to open a stuck dresser drawer.

correct answer 3. The pain has not been relieved by rest and nitroglycerin tablets. Rationale: The pain of angina may radiate to the left shoulder, arm, neck, or jaw. It is often precipitated by exertion or stress, has few associated symptoms, and is relieved by rest and nitroglycerin. The pain of MI may also radiate to the left arm, shoulder, jaw, and neck. It typically begins spontaneously, lasts longer than 30 minutes, and is frequently accompanied by associated symptoms (such as nausea, vomiting, dyspnea, diaphoresis, or anxiety). The pain of MI is not relieved by rest and nitroglycerin and requires opioid analgesics such as morphine sulfate for relief.

A client has just returned from the cardiac catheterization laboratory. The left femoral vessel was used as the access site. After returning the client to bed and conducting an initial assessment, the nurse assisting in caring for the client expects the cardiologist to write a prescription for the client to remain on bed rest. In which position would the bed be positioned? 1. In the high-Fowler's position 2. With the head of bed elevated at least 60 degrees 3. With the head of bed elevated no more than 30 degrees 4. With the foot of bed elevated as much as tolerated by the client

correct answer 3. With the head of bed elevated no more than 30 degrees Rationale: Following cardiac catheterization, the extremity in which the catheter was inserted is kept straight for 4 to 6 hours. If the femoral artery was used, strict bed rest is enforced for 6 to 12 hours or per agency procedure. The client may turn from side to side. The affected leg is kept straight, and the head is elevated no more than 30 degrees until hemostasis is adequately achieved. The cardiologist's prescription for positioning is always followed. Some cardiologists may prescribe a supine position.

A client has just returned from the cardiac catheterization laboratory. The left femoral vessel was used as the access site. After returning the client to bed, the nurse places a sign above the bed stating that the client would remain on bed rest and in which position? 1. In semi-Fowler's position 2. With the head of the bed elevated 45 degrees 3. With the head of the bed elevated no more than 15 degrees 4. With the foot of the bed elevated as much as tolerated by the client

correct answer 3. With the head of the bed elevated no more than 15 degrees Rationale: Following cardiac catheterization, the extremity used for catheter insertion is kept straight for 4 to 6 hours. If the femoral artery was used, strict bed rest is necessary for 4 to 6 hours. The client may turn from side to side. The head of the bed is not elevated more than 15 degrees to prevent kinking of the blood vessel at the groin and possible arterial occlusion. The cardiologist's prescription for positioning is always followed. Some cardiologists may prescribe a supine position.

The nurse is reinforcing diet teaching for a client on a low-sodium diet for hypertension. The nurse determines that there is a need for further teaching when the client makes which statement? 1. "This diet will help lower my blood pressure." 2. "The reason I need lower salt intake is to reduce fluid retention." 3. "This diet is not a replacement for my antihypertensive medications." 4. "Canned foods are inexpensive and are good to use on a low-sodium diet."

correct answer 4. "Canned foods are inexpensive and are good to use on a low-sodium diet." Rationale: A low-sodium diet is used as an adjunct to antihypertensive medications for the treatment of hypertension. Any commercial food that contains preservative is a significant source of sodium. Sodium retains fluid, which leads to hypertension secondary to increased fluid volume. Canned foods use salt as a preservative and should not be encouraged as part of a low-sodium diet. Lifelong medication is necessary in the treatment of hypertension.

A client receiving total parenteral nutrition (TPN) has a history of heart failure. The primary health care provider has prescribed furosemide 40 mg orally daily to prevent fluid overload. The nurse is giving instructions about taking furosemide in relation to the client's health plan. Which statement by the client indicates a need for further teaching? 1. "I need to get up slowly from my recliner." 2. "I need to take it first thing in the morning." 3. "I need to eat more foods high in potassium." 4. "I need to talk to my doctor about increasing my digoxin."

correct answer 4. "I need to talk to my doctor about increasing my digoxin." Rationale: Furosemide is a potassium-losing diuretic. Instructions include a need for a high-potassium diet or potassium replacement, rising slowly from a lying or sitting position because orthostatic hypotension may occur, and taking the medication early in the day to prevent sleeplessness and nocturia. Furosemide can increase the risk of toxicity of lithium, nondepolarizing skeletal muscle relaxants, digoxin, salicylates, aminoglycosides, and cisplatin.

The nurse is talking with a client with angina about factors that can precipitate an angina attack. Which statement by the client indicates an understanding of the precipitating events? 1. "I am going to run a mile each day." 2. "I am going to switch to electronic cigarettes." 3. "I will walk up two flights of stairs without stopping." 4. "I will pay my neighbor to shovel my snow this winter."

correct answer 4. "I will pay my neighbor to shovel my snow this winter." Rationale: Excessive exertion and cold frequently trigger angina attacks. Having the neighbor shovel snow will prevent the client from exertion in cold weather. Electronic cigarettes contain nicotine, which causes vasoconstriction.

A client is taking nicotinic acid for hyperlipidemia, and the nurse reinforces instructions to the client about the medication. Which statement by the client indicates an understanding of the instructions? 1. "It is not necessary to avoid the use of alcohol." 2. "The medication should be taken with meals to decrease flushing." 3. "Clay-colored stools are a common side effect and should not be of concern." 4. "Ibuprofen taken 30 minutes before the nicotinic acid should decrease the flushing."

correct answer 4. "Ibuprofen taken 30 minutes before the nicotinic acid should decrease the flushing." Rationale: Flushing is a side effect of this medication. Aspirin or a nonsteroidal antiinflammatory drug can be taken 30 minutes before taking the medication to decrease flushing. Alcohol consumption needs to be avoided because it will enhance this side effect. The medication should be taken with meals; this will decrease gastrointestinal upset. Taking the medication with meals has no effect on the flushing. Clay-colored stools are a sign of hepatic dysfunction and should be immediately reported to the PHCP.

The nurse is discussing smoking cessation with a client diagnosed with coronary artery disease (CAD). Which statement would the nurse make to the client to try to motivate the client to quit smoking? 1. "Since the damage has already been done, it will be all right to cut down a little at a time." 2. "None of the cardiovascular effects are reversible, but quitting might prevent lung cancer." 3. "If you totally quit smoking right now, you can cut your cardiovascular risk to zero within a year." 4. "If you quit now, your risk of cardiovascular disease will decrease to that of a nonsmoker in 3 to 4 years."

correct answer 4. "If you quit now, your risk of cardiovascular disease will decrease to that of a nonsmoker in 3 to 4 years." Rationale: The risks to the cardiovascular system from smoking are noncumulative and are not permanent. Three to 4 years after cessation, a client's cardiovascular risk is comparable to that of a person who never smoked. Therefore, options 1, 2, and 3 are incorrect.

A client with coronary artery disease has selected guided imagery to help cope with psychological stress. Which statement by the client indicates understanding of this stress reduction measure? 1. "This will help only if I play music at the same time." 2. "This will work for me only if I am alone in a quiet area." 3. "I need to do this only when I lie down in case I fall asleep." 4. "The best thing about this is that I can use it anywhere, anytime."

correct answer 4. "The best thing about this is that I can use it anywhere, anytime." Rationale: Guided imagery involves the client's creation of an image in the mind, concentrating on the image, and gradually becoming less aware of the offending stimulus. It does not require any adjuncts and does not need to be done in a quiet area only, although some clients may use other relaxation techniques or play music with it.

The nurse is reinforcing instructions to a client with essential hypertension about medication therapy with irbesartan. Which client statement would indicate a need for further teaching? 1. "I will take the medication each morning." 2. "I should stop smoking and drinking caffeine." 3. "I will monitor my blood pressure frequently." 4. "The medication reduces my need for exercise."

correct answer 4. "The medication reduces my need for exercise." Rationale: The medication irbesartan is an antihypertensive, and as with any antihypertensive, the client must maintain a healthy lifestyle that includes dietary modifications and exercise. Antihypertensives should be taken in the morning. Smoking and consuming caffeine must be avoided. The client should be taught how to monitor his own blood pressure.

A client is experiencing impotence after taking an antihypertensive medication. The client states, "I would sooner have a stroke than keep living with the side effects of this medication." The nurse would make which appropriate response to the client? 1. "I can understand completely." 2. "You wouldn't really want to have a stroke." 3. "That health care provider should change your prescription." 4. "You are concerned about the side effects of your medication?"

correct answer 4. "You are concerned about the side effects of your medication?" Rationale: Reflection of the client's comment lets the client know that you are hearing his concerns without judging. The nurse cannot understand what the client is experiencing (option 1). Option 3 devalues the health care provider's judgment. Option 2 is confrontational and unsupportive.

A client had an aortic valve replacement 2 days ago. This morning, the client tells the nurse, "I don't feel any better than I did before surgery." Which response by the nurse is most appropriate? 1. "You will feel better in a week or two." 2. "It's only the second day postop. Cheer up." 3. "This is a normal frustration. It'll get better." 4. "You are concerned that you don't feel any better after surgery?"

correct answer 4. "You are concerned that you don't feel any better after surgery?" Rationale: Paraphrasing is restating the client's message in the nurse's own words. Paraphrasing may be in the form of a question. Option 4 uses the therapeutic communication technique of paraphrasing. The client is frustrated and is searching for understanding. Options 1, 2, and 3 are inappropriate communication techniques. Option 1 belittles the client's concerns. Options 2 and 3 offer false reassurance by the nurse.

The nurse is providing cardiopulmonary resuscitation (CPR) to an adult cardiac arrest victim. Which is the proper compression-to-ventilation ratio for one-person CPR? 1. 15:2 2. 15:4 3. 30:1 4. 30:2

correct answer 4. 30:2 Rationale: Current cardiopulmonary resuscitation guidelines based on evidence-based practice for one-person cardiopulmonary resuscitation recommend a 30 compression:2 respiration ratio. All other options are incorrect and will be less effective in the resuscitation of this victim.

The nurse is checking the apical heart rate of a client with a complaint of angina. The nurse places the stethoscope in which anatomical area? Refer to figure. 1. 1 2. 2 3. 3 4. 4

correct answer 4. 4 Rationale: The apical heart rate is assessed best by placing the stethoscope in the mitral area, which is located in the fifth intercostal space on the left side of the chest at the apex of the heart. The aortic area is located in the second intercostal space just right of the sternum. Erb's point is located in the third intercostal space just left of the sternum. The pulmonic area is located in the second intercostal space just left of the sternum.

The nurse has provided discharge instructions to a client being placed on long-term anticoagulant therapy with warfarin sodium. The nurse reminds the client to do which? 1. Reduce alcohol intake to 12 ounces daily. 2. Alternate the timing of the daily dose. 3. Take any over-the-counter medications as needed. 4. Avoid taking products containing acetylsalicylic acid.

correct answer 4. Avoid taking products containing acetylsalicylic acid. Rationale: Warfarin sodium is an anticoagulant. The client should avoid taking aspirin because of its antiplatelet properties and should avoid taking other over-the-counter medications without checking with the primary health care provider first because they could contain ingredients that would interact with the warfarin sodium. The client should avoid alcohol while taking warfarin sodium per primary health care provider's directions. The client should take the medication at the same time each day to increase compliance and keep therapeutic blood levels stable.

The nurse is beginning to ambulate a client with activity intolerance caused by bacterial endocarditis. The nurse determines that the client is best tolerating ambulation if which parameter is noted? 1. Mild dyspnea after walking 10 feet 2. Minimal chest pain rated 1 on a 1-to-10 pain scale 3. Pulse rate that increases from 68 to 94 beats per minute 4. Blood pressure that increases from 114/82 to 118/86 mm Hg

correct answer 4. Blood pressure that increases from 114/82 to 118/86 mm Hg Rationale: General indicators that a client is tolerating exercise include an absence of chest pain or dyspnea, a pulse rate increase of less than 20 beats per minute, and a blood pressure change of less than 10 mm Hg.

An automatic external defibrillator (AED) interprets that the rhythm of a pulseless client is ventricular fibrillation. The nurse takes which action next? 1. Administers rescue breathing during the defibrillation 2. Performs cardiopulmonary resuscitation for 1 minute before defibrillating 3. Charges the machine and immediately pushes the discharge buttons on the console 4. Orders personnel away from the client, charges the machine, and depresses the discharge buttons

correct answer 4. Orders personnel away from the client, charges the machine, and depresses the discharge buttons Rationale: If the AED advises to defibrillate, the rescuer orders all personnel away from the client, charges the machine, and pushes both of the discharge buttons on the console at the same time. The charge is delivered through the patch electrodes, so this method is known as "hands off" defibrillation, which is safer for the rescuer. The sequence of charges (up to three consecutive attempts at 200, 300, and 360 joules) is similar to that of conventional defibrillation.

A licensed practical nurse (LPN) assisting a registered nurse in the cardiac care unit (CCU) prepares to admit a client with a diagnosis of myocardial infarction (MI). The LPN would be certain to have which item(s) readily available on the unit when the client arrives by stretcher? 1. Trapeze bar 2. Bedside commode 3. Electrocardiogram machine 4. Oxygen cannula and flowmeter

correct answer 4. Oxygen cannula and flowmeter Rationale: The client will require oxygen therapy following MI to decrease pain because of ischemia and to help minimize the risk of its recurrence. Oxygen decreases pain to the myocardium by improving oxygen supply. An electrocardiogram machine and bedside commode may be used but are not priorities at this time. An overhead trapeze could actually be harmful by causing the client to use the Valsalva maneuver while pulling up in bed.

The nurse is working with a client who has been diagnosed with Prinzmetal's (variant) angina. The nurse plans to reinforce which information about this type of angina when teaching the client? 1. Prinzmetal's angina is effectively managed by beta-blocking agents. 2. Prinzmetal's angina improves with a low-sodium, high-potassium diet. 3. Prinzmetal's angina has the same risk factors as stable and unstable angina. 4. Prinzmetal's angina is generally treated with calcium channel blocking agents.

correct answer 4. Prinzmetal's angina is generally treated with calcium channel blocking agents. Rationale: Prinzmetal's angina results from spasm of the coronary arteries and is generally treated with calcium channel blocking agents. The risk factors are unknown, and this type of angina is relatively unresponsive to nitrates. Beta blockers are contraindicated because they may actually worsen the spasm. Diet therapy is not specifically indicated, although a healthy diet consuming foods low in fat and sodium is advocated in cardiac disease.

Due to an extreme staff shortage, the nurse has been sent to the intensive care unit to assist registered nurses in the care of clients. The nurse understands that which factor is most important to consider when treating a client with cardiogenic shock? 1. Diuretics to decrease circulating volume 2. Whole blood has lowest risk for restoring fluid volume 3. Intravenous and oral fluids are used to restore circulating volume 4. Restriction of volume expanders because of secondary pulmonary edema

correct answer 4. Restriction of volume expanders because of secondary pulmonary edema Rationale: Because there are several types of shock, it is important to know which shock state the client is in so that it can be managed appropriately. A client in cardiogenic shock may have secondary issues such as pulmonary edema due to the ineffective pumping mechanism of the heart. Fluid restoration is a key ingredient in treating shock states but must be used conservatively in clients with cardiogenic shock.

The nurse is assigned to assist with caring for a client after cardiac catheterization performed through the left femoral artery. The nurse would plan to maintain bed rest for this client in which position? 1. High-Fowler's position 2. Supine with no head elevation 3. Left lateral (side-lying) position 4. Supine with head elevation no greater than 30 degrees

correct answer 4. Supine with head elevation no greater than 30 degrees Rationale: After cardiac catheterization, the extremity into which the catheter was inserted is kept straight for the prescribed time period to prevent arterial occlusion or bleeding and hematoma. With a femoral approach, the client's affected extremity is kept straight and the head elevated no more than 30 degrees (some cardiologists prefer a lower head position or the flat position) until hemostasis is adequately achieved. The client may turn from side to side. Bathroom privileges are not allowed during the immediate postcatheterization period. High-Fowler's (90-degree elevation), flat, and side lying on the puncture site are not effective in preventing complications or allowing for client comfort.

A client has an Unna boot applied for treatment of a venous stasis leg ulcer. The nurse notes that the client's toes are mottled and cool, and the client verbalizes some numbness and tingling of the foot. Which interpretation would the nurse make of these findings? 1. The boot has not yet dried. 2. The boot is controlling leg edema. 3. The boot is impairing venous return. 4. The boot has been applied too tightly.

correct answer 4. The boot has been applied too tightly. Rationale: An Unna boot that is applied too tightly can cause signs of arterial occlusion. The nurse assesses the circulation in the foot and teaches the client to do the same. The other options are incorrect interpretations.

A client with a diagnosis of heart failure (HF) is preparing for discharge to home from the hospital. Which condition indicates the client is ready for discharge to home? 1. The client can get the prescriptions filled. 2. The client can be self-sufficient at home without any help. 3. The client can independently dress and put on support hose. 4. The client can verbally describe the daily medications, doses, and times to be administered.

correct answer 4. The client can verbally describe the daily medications, doses, and times to be administered Rationale: Medication therapy is an essential part of the therapeutic regimen for treating heart failure. The client must have a clear understanding of which medications to take and when. Options 1 and 3 can be carried out with the assistance of someone else. Option 2 may not be realistic for this client.

A client is recovering following a cardiac arrest. Which findings would the nurse act on first? 1. The client's urinary output is 40 mL/hr. 2. The client's temperature is normothermic. 3. The client's oxygen saturation is maintained at 95%. 4. The client's respiratory rate is 28 breaths per minute.

correct answer 4. The client's respiratory rate is 28 breaths per minute. Rationale: Post-cardiac care focuses on adequate tissue perfusion. Continuous assessment of the client's vital signs and clinical presentation is essential to make minute changes that will optimize their recovery process and maintain tissue perfusion. The client's respiratory rate represents hyperventilation. Hyperventilation will lead to decreased carbon dioxide (CO2) levels in the blood, which can lead to complications during the post-cardiac arrest recovery

A client who is 36 hours post-myocardial infarction has ambulated for the first time. The nurse determines that the client best tolerated the activity if which observation is made? 1. The skin is cool but slightly diaphoretic. 2. Dyspnea is noted only at the end of the exercise. 3. The preactivity blood pressure (BP) is 140/84 mm Hg; the postactivity BP is 110/72 mm Hg. 4. The preactivity pulse rate is 86 beats per minute; the postactivity pulse rate is 94 beats per minute.

correct answer 4. The preactivity pulse rate is 86 beats per minute; the postactivity pulse rate is 94 beats per minute. Rationale: The nurse checks vital signs and the level of fatigue with each activity. The client is not tolerating the activity if systolic BP drops more than 20 mm Hg, if the pulse rate increases more than 20 beats per minute, or if the client experiences dyspnea or chest pain. In addition, a significant drop in BP can indicate orthostatic hypotension, which is an abnormal condition. Cool, diaphoretic skin is a sign of some degree of cardiovascular compromise.

An unconscious client, bleeding profusely, is brought to the emergency department after a serious accident. Surgery is required immediately to save the client's life. With regard to informed consent for the surgical procedure, which is the best action? 1. Call the nursing supervisor to initiate a court order for the surgical procedure. 2. Try calling the client's spouse to obtain telephone consent before the surgical procedure. 3. Ask the friend who accompanied the client to the emergency department to sign the consent form. 4. Transport the client to the operating department immediately without obtaining an informed consent.

correct answer 4. Transport the client to the operating department immediately without obtaining an informed consent. Rationale: Generally there are only two instances in which the informed consent of an adult client is not needed. One instance is when an emergency is present and delaying treatment for the purpose of obtaining informed consent would result in injury or death to the client. The second instance is when the client waives the right to give informed consent. Options 1, 2, and 3 are inappropriate.

A licensed practical nurse (LPN) is a certified basic life support (BLS) instructor. The LPN is conducting a BLS recertification class and is discussing automated external defibrillation. A member of the class asks the LPN to identify the correct location for the placement of conductive gel pads to treat ventricular fibrillation. The LPN tells the class that the conductive gel pads are placed in which location on the client's chest? 1. Bilaterally under the right and left clavicles 2. Parallel between the umbilicus and the left nipple 3. Centered on the upper and lower half of the sternum 4. Under the right clavicle and to the left of the precordium

correct answer 4. Under the right clavicle and to the left of the precordium Rationale: In defibrillation, one gel pad is placed on the upper right chest next to the sternum and below the clavicle and the other to the left of the precordium. The electrode paddles are placed over the pads for defibrillation with firm pressure. Options 1, 2, and 3 are incorrect.


Related study sets

Ratio 1 and 2, Unit Rates and percents 2023

View Set

Chapter 45: Management of Patients With Oral and Esophageal Disorders

View Set

US Health Care CHPT. 4-6 (EXAM 2 Study)

View Set

Economics - Principle of Economics

View Set

MTTC1: Elem Ed. (Website Practice Test)

View Set

Systematic Reviews & Meta-Analyses

View Set

Physical Science Final Exam Part 2

View Set

Chapter 2: Administration of Drugs - PrepU

View Set